NUR 113: Intrapartum- Test 2

¡Supera tus tareas y exámenes ahora con Quizwiz!

As the United States and Canada continue to become more culturally diverse, it is increasingly important for the nursing staff to recognize a wide range of varying cultural beliefs and practices. Nurses need to develop respect for these culturally diverse practices and learn to incorporate these into a mutually agreed on plan of care. Although it is common practice in the United States for the father of the baby to be present at the birth, in many societies this is not the case. When implementing care, the nurse would anticipate that a woman from which country would have the father of the baby in attendance? a. Mexico c. Iran b. China d. India

ANS:A A woman from Mexico may be stoic about discomfort until the second stage, at which time she will request pain relief. Fathers and female relatives are usually in attendance during the second stage of labor. The father of the baby is expected to provide encouragement, support, and reassurance that all will be well. Fathers are usually not present in China. The Iranian father will not be present. Female support persons and female care providers are preferred. For many, a male caregiver is unacceptable. The father is usually not present in India, but female relatives are usually present. Natural childbirth methods are preferred.

Nursing care measures are commonly offered to women in labor. Which nursing measure reflects application of the gate-control theory? a. Massaging the woman's back b. Changing the woman's position c. Giving the prescribed medication d. Encouraging the woman to rest between contractions

ANS:A According to the gate-control theory, pain sensations travel along sensory nerve pathways to the brain, but only a limited number of sensations, or messages, can travel through these nerve pathways at one time. Distraction techniques such as massage or stroking, music, focal points, and imagery reduce or completely block the capacity of nerve pathways to transmit pain. These distractions are thought to work by closing down a hypothetic gate in the spinal cord and thus preventing pain signals from reaching the brain. The perception of pain is thereby diminished. Changing the woman's position, giving prescribed medication, and encouraging rest do not reduce or block the capacity of nerve pathways to transmit pain using the gate-control theory.

The nurse providing care for the laboring woman should understand that amnioinfusion is used to treat: a. Variable decelerations. c. Fetal bradycardia. b. Late decelerations. d. Fetal tachycardia.

ANS:A Amnioinfusion is used during labor either to dilute meconium-stained amniotic fluid or to supplement the amount of amniotic fluid to reduce the severity of variable decelerations caused by cord compression. Amnioinfusion has no bearing on late decelerations, fetal bradycardia, or fetal tachycardia alterations in fetal heart rate (FHR) tracings.

The nurse providing newborn stabilization must be aware that the primary side effect of maternal narcotic analgesia in the newborn is: a. Respiratory depression. c. Acrocyanosis. b. Bradycardia. d. Tachypnea.

ANS:A An infant delivered within 1 to 4 hours of maternal analgesic administration is at risk for respiratory depression from the sedative effects of the narcotic. Bradycardia is not the anticipated side effect of maternal analgesics. Acrocyanosis is an expected finding in a newborn and is not related to maternal analgesics. The infant who is having a side effect to maternal analgesics normally would have a decrease in respirations, not an increase.

A woman in preterm labor at 30 weeks of gestation receives two 12-mg doses of betamethasone intramuscularly. The purpose of this pharmacologic treatment is to: a. Stimulate fetal surfactant production. b. Reduce maternal and fetal tachycardia associated with ritodrine administration. c. Suppress uterine contractions. d. Maintain adequate maternal respiratory effort and ventilation during magnesium sulfate therapy.

ANS:A Antenatal glucocorticoids given as intramuscular injections to the mother accelerate fetal lung maturity. Inderal would be given to reduce the effects of ritodrine administration. Betamethasone has no effect on uterine contractions. Calcium gluconate would be given to reverse the respiratory depressive effects of magnesium sulfate therapy.

Which occurrence is associated with cervical dilation and effacement? a. Bloody show c. Lightening b. False labor d. Bladder distention

ANS:A As the cervix begins to soften, dilate, and efface, expulsion of the mucous plug that sealed the cervix during pregnancy occurs. This causes rupture of small cervical capillaries. Cervical dilation and effacement do not occur with false labor. Lightening is the descent of the fetus toward the pelvic inlet before labor. Bladder distention occurs when the bladder is not emptied frequently. It may slow down the descent of the fetus during labor.

With regard to the process of augmentation of labor, the nurse should be aware that it: a. Is part of the active management of labor that is instituted when the labor process is unsatisfactory. b. Relies on more invasive methods when oxytocin and amniotomy have failed. c. Is a modern management term to cover up the negative connotations of forceps-assisted birth. d. Uses vacuum cups.

ANS:A Augmentation is part of the active management of labor that stimulates uterine contractions after labor has started but is not progressing satisfactorily. Augmentation uses amniotomy and oxytocin infusion, as well as some gentler, noninvasive methods. Forceps-assisted births and vacuum-assisted births are appropriately used at the end of labor and are not part of augmentation.

With regard to nerve block analgesia and anesthesia, nurses should be aware that: a. Most local agents are related chemically to cocaine and end in the suffix -caine. b. Local perineal infiltration anesthesia is effective when epinephrine is added, but it can be injected only once. c. A pudendal nerve block is designed to relieve the pain from uterine contractions. d. A pudendal nerve block, if done correctly, does not significantly lessen the bearing-down reflex.

ANS:A Common agents include lidocaine and chloroprocaine. Injections can be repeated to prolong the anesthesia. A pudendal nerve block relieves pain in the vagina, vulva, and perineum but not the pain from uterine contractions, and it lessens or shuts down the bearing-down reflex.

Which patient status is an acceptable indication for serial oxytocin induction of labor? a. Past 42 weeks' gestation c. Polyhydramnios b. Multiple fetuses d. History of long labors

ANS:A Continuing a pregnancy past the normal gestational period is likely to be detrimental to fetal health. Multiple fetuses overdistend the uterus and make induction of labor high risk. Polyhydramnios overdistends the uterus, again making induction of labor high risk. History of rapid labors is a reason for induction of labor because of the possibility that the baby would otherwise be born in uncontrolled circumstances.

A normal uterine activity pattern in labor is characterized by: a. Contractions every 2 to 5 minutes. b. Contractions lasting about 2 minutes. c. Contractions about 1 minute apart. d. A contraction intensity of about 1000 mm Hg with relaxation at 50 mm Hg.

ANS:A Contractions normally occur every 2 to 5 minutes and last less than 90 seconds (intensity 800 mm Hg) with about 30 seconds in between (20 mm Hg or less).

A woman is experiencing back labor and complains of intense pain in her lower back. An effective relief measure would be to use: a. Counterpressure against the sacrum. b. Pant-blow (breaths and puffs) breathing techniques. c. Effleurage. d. Conscious relaxation or guided imagery.

ANS:A Counterpressure is steady pressure applied by a support person to the sacral area with the fist or heel of the hand. This technique helps the woman cope with the sensations of internal pressure and pain in the lower back. The pain management techniques of pant-blow, effleurage, and conscious relaxation or guided imagery are usually helpful for contractions per the gate-control theory.

The nurse would expect which maternal cardiovascular finding during labor? a. Increased cardiac output b. Decreased pulse rate c. Decreased white blood cell (WBC) count d. Decreased blood pressure

ANS:A During each contraction, 400 mL of blood is emptied from the uterus into the maternal vascular system. This increases cardiac output by about 51% above baseline pregnancy values at term. The heart rate increases slightly during labor. The WBC count can increase during labor. During the first stage of labor, uterine contractions cause systolic readings to increase by about 10 mm Hg. During the second stage, contractions may cause systolic pressures to increase by 30 mm Hg and diastolic readings to increase by 25 mm Hg.

Which deceleration of the fetal heart rate would not require the nurse to change the maternal position? a. Early decelerations b. Late decelerations c. Variable decelerations d. It is always a good idea to change the woman's position.

ANS:A Early decelerations (and accelerations) generally do not need any nursing intervention. Late decelerations suggest that the nurse should change the maternal position (lateral); variable decelerations also require a maternal position change (side to side). Although changing positions throughout labor is recommended, it is not required in response to early decelerations.

The nurse caring for the laboring woman should understand that early decelerations are caused by: a. Altered fetal cerebral blood flow. c. Uteroplacental insufficiency. b. Umbilical cord compression. d. Spontaneous rupture of membranes.

ANS:A Early decelerations are the fetus's response to fetal head compression. Variable decelerations are associated with umbilical cord compression. Late decelerations are associated with uteroplacental insufficiency. Spontaneous rupture of membranes has no bearing on the fetal heart rate unless the umbilical cord prolapses, which would result in variable or prolonged bradycardia.

The nurse providing care for the laboring woman comprehends that accelerations with fetal movement: a. Are reassuring. b. Are caused by umbilical cord compression. c. Warrant close observation. d. Are caused by uteroplacental insufficiency.

ANS:A Episodic accelerations in the fetal heart rate (FHR) occur during fetal movement and are indications of fetal well-being. Umbilical cord compression results in variable decelerations in the FHR. Accelerations in the FHR are an indication of fetal well-being and do not warrant close observation. Uteroplacental insufficiency would result in late decelerations in the FHR.

Fetal well-being during labor is assessed by: a. The response of the fetal heart rate (FHR) to uterine contractions (UCs). b. Maternal pain control. c. Accelerations in the FHR. d. An FHR above 110 beats/min.

ANS:A Fetal well-being during labor can be measured by the response of the FHR to UCs. In general, reassuring FHR patterns are characterized by an FHR baseline in the range of 110 to 160 beats/min with no periodic changes, a moderate baseline variability, and accelerations with fetal movement. Maternal pain control is not the measure used to determine fetal well-being in labor. Although FHR accelerations are a reassuring pattern, they are only one component of the criteria by which fetal well-being is assessed. Although an FHR above 110 beats/min may be reassuring, it is only one component of the criteria by which fetal well-being is assessed. More information would be needed to determine fetal well-being.

With regard to breathing techniques during labor, maternity nurses should understand that: a. Breathing techniques in the first stage of labor are designed to increase the size of the abdominal cavity to reduce friction. b. By the time labor has begun, it is too late for instruction in breathing and relaxation. c. Controlled breathing techniques are most difficult near the end of the second stage of labor. d. The patterned-paced breathing technique can help prevent hyperventilation.

ANS:A First-stage techniques promote relaxation of abdominal muscles, thereby increasing the size of the abdominal cavity. Instruction in simple breathing and relaxation techniques early in labor is possible and effective. Controlled breathing techniques are most difficult in the transition phase at the end of the first stage of labor when the cervix is dilated 8 to 10 cm. Patterned-paced breathing sometimes can lead to hyperventilation.

Which description of the four stages of labor is correct for both definition and duration? a. First stage: onset of regular uterine contractions to full dilation; less than 1 hour to 20 hours b. Second stage: full effacement to 4 to 5 cm; visible presenting part; 1 to 2 hours c. Third state: active pushing to birth; 20 minutes (multiparous women), 50 minutes (first-timer) d. Fourth stage: delivery of the placenta to recovery; 30 minutes to 1 hour

ANS:A Full dilation may occur in less than 1 hour, but in first-time pregnancies it can take up to 20 hours. The second stage extends from full dilation to birth and takes an average of 20 to 50 minutes, although 2 hours is still considered normal. The third stage extends from birth to expulsion of the placenta and usually takes a few minutes. The fourth stage begins after expulsion of the placenta and lasts until homeostasis is reestablished (about 2 hours).

What is an essential part of nursing care for the laboring woman? a. Helping the woman manage the pain b. Eliminating the pain associated with labor c. Sharing personal experiences regarding labor and delivery to decrease her anxiety d. Feeling comfortable with the predictable nature of intrapartum care

ANS:A Helping a woman manage the pain is an essential part of nursing care because pain is an expected part of normal labor and cannot be fully relieved. Decreasing anxiety is important; however, managing pain is a top priority. The labor nurse should consistently deliver care based on the standard of care related to the maternity patient. Because of the unpredictable nature of labor, the nurse should always be alert for unanticipated events.

Which presentation is described accurately in terms of both presenting part and frequency of occurrence? a. Cephalic: occiput; at least 95% c. Shoulder: scapula; 10% to 15% b. Breech: sacrum; 10% to 15% d. Cephalic: cranial; 80% to 85%

ANS:A In cephalic presentations (head first), the presenting part is the occiput; this occurs in 96% of births. In a breech birth, the sacrum emerges first; this occurs in about 3% of births. In shoulder presentations, the scapula emerges first; this occurs in only 1% of births.

Which maternal condition is considered a contraindication for the application of internal monitoring devices? a. Unruptured membranes c. External monitors in current use b. Cervix dilated to 4 cm d. Fetus with a known heart defect

ANS:A In order to apply internal monitoring devices, the membranes must be ruptured. Cervical dilation of 4 cm permits the insertion of fetal scalp electrodes and intrauterine catheter. The external monitor can be discontinued after the internal ones are applied. A compromised fetus should be monitored with the most accurate monitoring devices.

While evaluating an external monitor tracing of a woman in active labor, the nurse notes that the fetal heart rate (FHR) for five sequential contractions begins to decelerate late in the contraction, with the nadir of the decelerations occurring after the peak of the contraction. The nurse's first priority is to: a. Change the woman's position. c. Assist with amnioinfusion. b. Notify the care provider. d. Insert a scalp electrode.

ANS:A Late decelerations may be caused by maternal supine hypotension syndrome. They usually are corrected when the woman turns on her side to displace the weight of the gravid uterus from the vena cava. If the fetus does not respond to primary nursing interventions for late decelerations, the nurse would continue with subsequent intrauterine resuscitation measures, including notifying the care provider. An amnioinfusion may be used to relieve pressure on an umbilical cord that has not prolapsed. The FHR pattern associated with this situation most likely reveals variable deceleration. A fetal scalp electrode would provide accurate data for evaluating the well-being of the fetus; however, this is not a nursing intervention that would alleviate late decelerations, nor is it the nurse's first priority.

Leopold maneuvers would be an inappropriate method of assessment to determine: a. Gender of the fetus. b. Number of fetuses. c. Fetal lie and attitude. d. Degree of the presenting part's descent into the pelvis.

ANS:A Leopold maneuvers help identify the number of fetuses, the fetal lie and attitude, and the degree of descent of the presenting part into the pelvis. The gender of the fetus is not a goal of the examination at this time.

The nurse caring for a laboring woman is aware that maternal cardiac output can be increased by: a. Change in position. c. Regional anesthesia. b. Oxytocin administration. d. Intravenous analgesic.

ANS:A Maternal supine hypotension syndrome is caused by the weight and pressure of the gravid uterus on the ascending vena cava when the woman is in a supine position. This reduces venous return to the woman's heart, as well as cardiac output, and subsequently reduces her blood pressure. The nurse can encourage the woman to change positions and avoid the supine position. Oxytocin administration, regional anesthesia, and intravenous analgesic may reduce maternal cardiac output.

A woman in active labor receives an analgesic opioid agonist. Which medication relieves severe, persistent, or recurrent pain; creates a sense of well-being; overcomes inhibitory factors; and may even relax the cervix but should be used cautiously in women with cardiac disease? a. Meperidine (Demerol) c. Butorphanol tartrate (Stadol) b. Promethazine (Phenergan) d. Nalbuphine (Nubain)

ANS:A Meperidine is the most commonly used opioid agonist analgesic for women in labor throughout the world. It overcomes inhibitory factors in labor and may even relax the cervix. Because tachycardia is a possible adverse reaction, meperidine is used cautiously in women with cardiac disease. Phenergan is an ataractic (tranquilizer) that may be used to augment the desirable effects of the opioid analgesics but has few of the undesirable effects of those drugs. Stadol and Nubain are opioid agonist-antagonist analgesics.

Perinatal nurses are legally responsible for: a. Correctly interpreting fetal heart rate (FHR) patterns, initiating appropriate nursing interventions, and documenting the outcomes. b. Greeting the client on arrival, assessing her, and starting an intravenous line. c. Applying the external fetal monitor and notifying the care provider. d. Making sure that the woman is comfortable.

ANS:A Nurses who care for women during childbirth are legally responsible for correctly interpreting FHR patterns, initiating appropriate nursing interventions based on those patterns, and documenting the outcomes of those interventions. Greeting the client, assessing her, and starting an IV; applying the external fetal monitor and notifying the care provider; and making sure the woman is comfortable may be activities that a nurse performs, but they are not activities for which the nurse is legally responsible.

When assessing a multiparous woman who has just given birth to an 8-pound boy, the nurse notes that the woman's fundus is firm and has become globular in shape. A gush of dark red blood comes from her vagina. The nurse concludes that: a. The placenta has separated. b. A cervical tear occurred during the birth. c. The woman is beginning to hemorrhage. d. Clots have formed in the upper uterine segment.

ANS:A Placental separation is indicated by a firmly contracting uterus, a change in the uterus from a discoid to a globular ovoid shape, a sudden gush of dark red blood from the introitus, an apparent lengthening of the umbilical cord, and a finding of vaginal fullness. Cervical tears that do not extend to the vagina result in minimal blood loss. Signs of hemorrhage are a boggy uterus, bright red vaginal bleeding, alterations in vital signs, pallor, lightheadedness, restlessness, decreased urinary output, and alteration in the level of consciousness. If clots have formed in the upper uterine segment, the nurse would expect to find the uterus boggy and displaced to the side.

Nurses should be aware of the differences experience can make in labor pain such as: a. Sensory pain for nulliparous women often is greater than for multiparous women during early labor. b. Affective pain for nulliparous women usually is less than for multiparous women throughout the first stage of labor. c. Women with a history of substance abuse experience more pain during labor. d. Multiparous women have more fatigue from labor and therefore experience more pain.

ANS:A Sensory pain is greater for nulliparous women because their reproductive tract structures are less supple. Affective pain is greater for nulliparous women during the first stage but decreases for both nulliparous and multiparous during the second stage. Women with a history of substance abuse experience the same amount of pain as those without such a history. Nulliparous women have longer labors and therefore experience more fatigue.

A means of controlling the birth of the fetal head with a vertex presentation is: a. The Ritgen maneuver. c. The lithotomy position. b. Fundal pressure. d. The De Lee apparatus.

ANS:A The Ritgen maneuver extends the head during the actual birth and protects the perineum. Gentle, steady pressure against the fundus of the uterus facilitates vaginal birth. The lithotomy position has been commonly used in Western cultures, partly because it is convenient for the health care provider. The De Lee apparatus is used to suction fluid from the infant's mouth.

In assessing a woman for pain and discomfort management during labor, a nurse most likely would: a. Have the woman use a visual analog scale (VAS) to determine her level of pain. b. Note drowsiness as a sign that the medications were working. c. Interpret a woman's fist clenching as an indication that she is angry at her male partner and the physician. d. Evaluate the woman's skin turgor to see whether she needs a gentle oil massage.

ANS:A The VAS is a means of adding the woman's assessment of her pain to the nurse's observations. Drowsiness is a side effect of medications, not usually (sedatives aside) a sign of effectiveness. The fist clenching likely is a sign of apprehension that may need attention. Skin turgor, along with the moistness of the membranes and the concentration of the urine, is a sign that helps the nurse evaluate hydration.

When using intermittent auscultation (IA) to assess uterine activity, the nurse should be cognizant that: a. The examiner's hand should be placed over the fundus before, during, and after contractions. b. The frequency and duration of contractions is measured in seconds for consistency. c. Contraction intensity is given a judgment number of 1 to 7 by the nurse and client together. d. The resting tone between contractions is described as either placid or turbulent.

ANS:A The assessment is done by palpation; duration, frequency, intensity, and resting tone must be assessed. The duration of contractions is measured in seconds; the frequency is measured in minutes. The intensity of contractions usually is described as mild, moderate, or strong. The resting tone usually is characterized as soft or relaxed.

The most critical nursing action in caring for the newborn immediately after birth is: a. Keeping the newborn's airway clear. b. Fostering parent-newborn attachment. c. Drying the newborn and wrapping the infant in a blanket. d. Administering eye drops and vitamin K.

ANS:A The care given immediately after the birth focuses on assessing and stabilizing the newborn. Although fostering parent-infant attachment is an important task for the nurse, it is not the most critical nursing action in caring for the newborn immediately after birth. The nursing activities would be (in order of importance) to maintain a patent airway, support respiratory effort, and prevent cold stress by drying the newborn and covering the infant with a warmed blanket or placing the newborn under a radiant warmer. After the newborn has been stabilized, the nurse assesses the newborn's physical condition, weighs and measures the newborn, administers prophylactic eye ointment and a vitamin K injection, affixes an identification bracelet, wraps the newborn in warm blankets, and then gives the infant to the partner or mother when he or she is ready.

The nurse would expect which maternal cardiovascular finding during labor? a. Increased cardiac output b. Decreased pulse rate c. Decreased white blood cell (WBC) count d. Decreased blood pressure

ANS:A The first stage of labor is considered to last from the onset of regular uterine contractions to full dilation of the cervix. The first stage is much longer than the second and third stages combined. In a first-time pregnancy the first stage of labor can take up to 20 hours. The second stage of labor lasts from the time the cervix is fully dilated to the birth of the fetus. The average length is 20 minutes for a multiparous woman and 50 minutes for a nulliparous woman. The third stage of labor lasts from the birth of the fetus until the placenta is delivered. This stage may be as short as 3 minutes or as long as 1 hour. The fourth stage of labor, recovery, lasts about 2 hours after delivery of the placenta.

To care for a laboring woman adequately, the nurse understands that the __________ stage of labor varies the most in length? a. First c. Third b. Second d. Fourth

ANS:A The first stage of labor is considered to last from the onset of regular uterine contractions to full dilation of the cervix. The first stage is much longer than the second and third stages combined. In a first-time pregnancy the first stage of labor can take up to 20 hours. The second stage of labor lasts from the time the cervix is fully dilated to the birth of the fetus. The average length is 20 minutes for a multiparous woman and 50 minutes for a nulliparous woman. The third stage of labor lasts from the birth of the fetus until the placenta is delivered. This stage may be as short as 3 minutes or as long as 1 hour. The fourth stage of labor, recovery, lasts about 2 hours after delivery of the placenta.

In evaluating the effectiveness of oxytocin induction, the nurse would expect: a. Contractions lasting 40 to 90 seconds, 2 to 3 minutes apart. b. The intensity of contractions to be at least 110 to 130 mm Hg. c. Labor to progress at least 2 cm/hr dilation. d. At least 30 mU/min of oxytocin will be needed to achieve cervical dilation.

ANS:A The goal of induction of labor would be to produce contractions that occur every 2 to 3 minutes and last 60 to 90 seconds. The intensity of the contractions should be 40 to 90 mm Hg by intrauterine pressure catheter. Cervical dilation of 1 cm/hr in the active phase of labor would be the goal in an oxytocin induction. The dose is increased by 1 to 2 mU/min at intervals of 30 to 60 minutes until the desired contraction pattern is achieved. Doses are increased up to a maximum of 20 to 40 mU/min.

Immediately after the forceps-assisted birth of an infant, the nurse should: a. Assess the infant for signs of trauma. b. Give the infant prophylactic antibiotics. c. Apply a cold pack to the infant's scalp. d. Measure the circumference of the infant's head.

ANS:A The infant should be assessed for bruising or abrasions at the site of application, facial palsy, and subdural hematoma. Prophylactic antibiotics are not necessary with a forceps delivery. A cold pack would put the infant at risk for cold stress and is contraindicated. Measuring the circumference of the head is part of the initial nursing assessment.

The exact cause of preterm labor is unknown and believed to be multifactorial. Infection is thought to be a major factor in many preterm labors. Select the type of infection that has not been linked to preterm births. a. Viral c. Cervical b. Periodontal d. Urinary tract

ANS:A The infections that increase the risk of preterm labor and birth are all bacterial. They include cervical, urinary tract, periodontal, and other bacterial infections. Therefore, it is important for the client to participate in early, continual, and comprehensive prenatal care. Evidence has shown a link between periodontal infections and preterm labor. Researchers recommend regular dental care before and during pregnancy, oral assessment as a routine part of prenatal care, and scrupulous oral hygiene to prevent infection. Cervical infections of a bacterial nature have been linked to preterm labor and birth. The presence of urinary tract infections increases the risk of preterm labor and birth.

In relation to primary and secondary powers, the maternity nurse comprehends that: a. Primary powers are responsible for effacement and dilation of the cervix. b. Effacement generally is well ahead of dilation in women giving birth for the first time; they are closer together in subsequent pregnancies. c. Scarring of the cervix caused by a previous infection or surgery may make the delivery a bit more painful, but it should not slow or inhibit dilation. d. Pushing in the second stage of labor is more effective if the woman can breathe deeply and control some of her involuntary needs to push, as the nurse directs.

ANS:A The primary powers are responsible for dilation and effacement; secondary powers are concerned with expulsion of the fetus. Effacement generally is well ahead of dilation in first-timers; they are closer together in subsequent pregnancies. Scarring of the cervix may slow dilation. Pushing is more effective and less fatiguing when the woman begins to push only after she has the urge to do so.

The nurse knows that proper placement of the tocotransducer for electronic fetal monitoring is located: a. Over the uterine fundus. c. Inside the uterus. b. On the fetal scalp. d. Over the mother's lower abdomen.

ANS:A The tocotransducer monitors uterine activity and should be placed over the fundus, where the most intensive uterine contractions occur. The tocotransducer is for external use.

When assessing a woman in the first stage of labor, the nurse recognizes that the most conclusive sign that uterine contractions are effective would be: a. Dilation of the cervix. c. Rupture of the amniotic membranes. b. Descent of the fetus. d. Increase in bloody show.

ANS:A The vaginal examination reveals whether the woman is in true labor. Cervical change, especially dilation, in the presence of adequate labor indicates that the woman is in true labor. Descent of the fetus, or engagement, may occur before labor. Rupture of membranes may occur with or without the presence of labor. Bloody show may indicate slow, progressive cervical change (e.g., effacement) in both true and false labor.

A pregnant woman's amniotic membranes rupture. Prolapsed umbilical cord is suspected. What intervention would be the top priority? a. Placing the woman in the knee-chest position b. Covering the cord in sterile gauze soaked in saline c. Preparing the woman for a cesarean birth d. Starting oxygen by face mask

ANS:A The woman is assisted into a position (e.g., modified Sims position, Trendelenburg position, or the knee-chest position) in which gravity keeps the pressure of the presenting part off the cord. Although covering the cord in sterile gauze soaked saline, preparing the woman for a cesarean, and starting oxygen by face mark are appropriate nursing interventions in the event of a prolapsed cord, the intervention of top priority would be positioning the mother to relieve cord compression.

In assisting with the two factors that have an effect on fetal status (i.e., pushing and positioning), nurses should: a. Encourage the woman's cooperation in avoiding the supine position. b. Advise the woman to avoid the semi-Fowler position. c. Encourage the woman to hold her breath and tighten her abdominal muscles to produce a vaginal response. d. Instruct the woman to open her mouth and close her glottis, letting air escape after the push.

ANS:A The woman should maintain a side-lying position. The semi-Fowler position is the recommended side-lying position with a lateral tilt to the uterus. The Valsalva maneuver, which encourages the woman to hold her breath and tighten her abdominal muscles, should be avoided. Both the mouth and glottis should be open, letting air escape during the push.

A laboring woman is lying in the supine position. The most appropriate nursing action at this time is to: a. Ask her to turn to one side. b. Elevate her feet and legs. c. Take her blood pressure. d. Determine whether fetal tachycardia is present.

ANS:A The woman's supine position may cause the heavy uterus to compress her inferior vena cava, thus reducing blood return to her heart and reducing placental blood flow. Elevating her legs will not relieve the pressure from the inferior vena cava. If the woman is allowed to stay in the supine position and blood flow to the placental is reduced significantly, fetal tachycardia may occur. The most appropriate nursing action is to prevent this from occurring by turning the woman to her side. Blood pressure readings may be obtained when the patient is in the appropriate and safest position.

To help clients manage discomfort and pain during labor, nurses should be aware that: a. The predominant pain of the first stage of labor is the visceral pain located in the lower portion of the abdomen. b. Referred pain is the extreme discomfort between contractions. c. The somatic pain of the second stage of labor is more generalized and related to fatigue. d. Pain during the third stage is a somewhat milder version of the second stage.

ANS:A This pain comes from cervical changes, distention of the lower uterine segment, and uterine ischemia. Referred pain occurs when the pain that originates in the uterus radiates to the abdominal wall, lumbosacral area of the back, iliac crests, and gluteal area. Second-stage labor pain is intense, sharp, burning, and localized. Third-stage labor pain is similar to that of the first stage.

. A pregnant woman is in her third trimester. She asks the nurse to explain how she can tell true labor from false labor. The nurse would explain that "true" labor contractions: a. Increase with activity such as ambulation. b. Decrease with activity. c. Are always accompanied by the rupture of the bag of waters. d. Alternate between a regular and an irregular pattern.

ANS:A True labor contractions become more intense with walking. False labor contractions often stop with walking or position changes. Rupture of membranes may occur before or during labor. True labor contractions are regular.

When managing the care of a woman in the second stage of labor, the nurse uses various measures to enhance the progress of fetal descent. These measures include: a. Encouraging the woman to try various upright positions, including squatting and standing. b. Telling the woman to start pushing as soon as her cervix is fully dilated. c. Continuing an epidural anesthetic so pain is reduced and the woman can relax. d. Coaching the woman to use sustained, 10- to 15-second, closed-glottis bearing-down efforts with each contraction.

ANS:A Upright positions and squatting both may enhance the progress of fetal descent. Many factors dictate when a woman will begin pushing. Complete cervical dilation is necessary, but it is only one factor. If the fetal head is still in a higher pelvic station, the physician or midwife may allow the woman to "labor down" (allowing more time for fetal descent, thereby reducing the amount of pushing needed) if the woman is able. The epidural may mask the sensations and muscle control needed for the woman to push effectively. Closed glottic breathing may trigger the Valsalva maneuver, which increases intrathoracic and cardiovascular pressures, reducing cardiac output and inhibiting perfusion of the uterus and placenta. In addition, holding the breath for longer than 5 to 7 seconds diminishes the perfusion of oxygen across the placenta and results in fetal hypoxia.

With regard to a woman's intake and output during labor, nurses should be aware that: a. The tradition of restricting the laboring woman to clear liquids and ice chips is being challenged because regional anesthesia is used more often than general anesthesia. b. Intravenous (IV) fluids usually are necessary to ensure that the laboring woman stays hydrated. c. Routine use of an enema empties the rectum and is very helpful for producing a clean, clear delivery. d. When a nulliparous woman experiences the urge to defecate, it often means birth will follow quickly.

ANS:A Women are awake with regional anesthesia and are able to protect their own airway, which reduces the worry over aspiration. Routine IV fluids during labor are unlikely to be beneficial and may be harmful. Routine use of an enema is at best ineffective and may be harmful. A multiparous woman may feel the urge to defecate and it may mean birth will follow quickly, but not for a first-timer.

Controlling pain is important to promoting wellness. Unrelieved pain has been associated with a. prolonged stress response and a cascade of harmful effects system-wide. b. large tidal volumes and decreased lung capacity. c. decreased tumor growth and longevity. d. decreased carbohydrate, protein, and fat destruction.

ANS:A, Pain triggers a number of physiologic stress responses in the human body. Unrelieved pain can prolong the stress response and produce a cascade of harmful effects in all body systems. The stress response causes the endocrine system to release excessive amounts of hormones, such as cortisol, catecholamines, and glucagon. Insulin and testosterone levels decrease. Increased endocrine activity in turn initiates a number of metabolic processes, in particular, accelerated carbohydrate, protein, and fat destruction, which can result in weight loss, tachycardia, increased respiratory rate, shock, and even death. The immune system is also affected by pain as demonstrated by research showing a link between unrelieved pain and a higher incidence of nosocomial infections and increased tumor growth. Large tidal volumes are not associated with pain while decreased lung capacity is associated with unrelieved pain. Decreased tumor growth and longevity are not associated with unrelieved pain. Decreased carbohydrate, protein, and fat are not associated with pain or stress response.

The nurse is admitting a prenatal patient for diagnostic testing. While eliciting the psychosocial history, the nurse learns the patient smokes a pack of cigarettes daily, drinks a cup of cappuccino with breakfast, has smoked weed in the remote past, and is a social drinker. Which action should the nurse first take? a. Strongly advise immediate tobacco cessation b. Elimination of all caffeinated beverages c. Serum and urine testing for drug use and alcohol use d. Referral to a 12-step program

ANS:A, There are numerous risk factors for women and men affecting reproductive health and pregnancy outcomes. These can be categorized into biophysical, psychosocial, sociodemographic, and environmental factors. Some of the risk factors for human reproduction fit into multiple categories. Psychosocial factors cover smoking, excessive caffeine, alcohol and drug abuse, psychologic status including impaired mental health, addictive lifestyles, spouse abuse, and noncompliance with cultural norms. Drinking a cup of a caffeinated beverage a day is not associated with adverse fetal outcomes usually. Serum and urine testing for drug/alcohol use is not required for stated marijuana use in the remote past. Patient referral to a 12-step program is usually advisable for current alcohol and/or drug use.

Women who have participated in childbirth education classes often bring a "birth bag" or "Lamaze bag" with them to the hospital. These items often assist in reducing stress and providing comfort measures. The nurse caring for women in labor should be aware of common items that a client may bring, including (Select all that apply): a. Rolling pin. d. Stuffed animal or photo. b. Tennis balls. e. Candles. c. Pillow.

ANS:A,B,C,D The rolling pin and tennis balls are used to provide counterpressure, especially if the woman is experiencing back labor. Although the facility has plenty of pillows, when the client brings her own, it is a reminder of home and provides added comfort. A stuffed animal or framed photo can be used to provide a focal point during contractions. Although many women find the presence of candles conducive to creating calm and relaxing surroundings, these are not suitable for a hospital birthing room environment. Oxygen may be in use, resulting in a fire hazard. Flameless candles are often sold in hospital gift shops. It is also important for the nurse to orient the patient and her family to the call bell and light switches to familiarize herself with the environment.

Which factors influence cervical dilation (Select all that apply) ? a. Strong uterine contractions b. The force of the presenting fetal part against the cervix c. The size of the female pelvis d. The pressure applied by the amniotic sac e. Scarring of the cervix

ANS:A,B,D,E Dilation of the cervix occurs by the drawing upward of the musculofibrous components of the cervix, which is caused by strong uterine contractions. Pressure exerted by the amniotic fluid while the membranes are intact or by the force applied by the presenting part also can promote cervical dilation. Scarring of the cervix as a result of a previous infection or surgery may slow cervical dilation. Pelvic size does not affect cervical dilation.

Signs that precede labor include (Select all that apply): a. Lightening. b. Exhaustion. c. Bloody show. d. Rupture of membranes. e. Decreased fetal movement.

ANS:A,C,D Signs that precede labor may include lightening, urinary frequency, backache, weight loss, surge of energy, bloody show, and rupture of membranes. Many women experience a burst of energy before labor. A decrease in fetal movement is an ominous sign that does not always correlate with labor.

Induction of labor is considered an acceptable obstetric procedure if it is in the best interest to deliver the fetus. The charge nurse on the labor and delivery unit is often asked to schedule patients for this procedure and therefore must be cognizant of the specific conditions appropriate for labor induction. These include (Select all that apply): a. Rupture of membranes at or near term. b. Convenience of the woman or her physician. c. Chorioamnionitis (inflammation of the amniotic sac). d. Post-term pregnancy. e. Fetal death.

ANS:A,C,D,E These are all acceptable indications for induction. Other conditions include intrauterine growth retardation (IUGR), maternal-fetal blood incompatibility, hypertension, and placental abruption. Elective inductions for the convenience of the woman or her provider are not recommended; however, they have become commonplace. Factors such as rapid labors and living a long distance from a health care facility may be valid reasons in such a circumstance. Elective delivery should not occur before 39 weeks' completed gestation.

The class of drugs known as opioid analgesics (butorphanol, nalbuphine) is not suitable for administration to women with known opioid dependence. The antagonistic activity could precipitate withdrawal symptoms (abstinence syndrome) in both mothers and newborns. Signs of opioid/narcotic withdrawal in the mother would include (Select all that apply): a. Yawning, runny nose. b. Increase in appetite. c. Chills and hot flashes. d. Constipation. e. Irritability, restlessness.

ANS:A,C,E The woman experiencing maternal opioid withdrawal syndrome will exhibit yawning, runny nose, sneezing, anorexia, chills or hot flashes, vomiting, diarrhea, abdominal pain, irritability, restlessness, muscle spasms, weakness, and drowsiness. It is important for the nurse to assess both mother and baby and to plan care accordingly.

In planning for an expected cesarean birth for a woman who has given birth by cesarean previously and who has a fetus in the transverse presentation, which information would the nurse include? a. "Because this is a repeat procedure, you are at the lowest risk for complications." b. "Even though this is your second cesarean birth, you may wish to review the preoperative and postoperative procedures." c. "Because this is your second cesarean birth, you will recover faster." d. "You will not need preoperative teaching because this is your second cesarean birth."

ANS:B "Even though this is your second cesarean birth, you may wish to review the preoperative and postoperative procedures" is the most appropriate statement. It is not accurate to state that the woman is at the lowest risk for complications. Both maternal and fetal risks are associated with every cesarean section. "Because this is your second cesarean birth, you will recover faster" is not an accurate statement. Physiologic and psychologic recovery from a cesarean section is multifactorial and individual to each client each time. Preoperative teaching should always be performed, regardless of whether the client has already had this procedure.

A woman who is 39 weeks pregnant expresses fear about her impending labor and how she will manage. The nurse's best response is: a. "Don't worry about it. You'll do fine." b. "It's normal to be anxious about labor. Let's discuss what makes you afraid." c. "Labor is scary to think about, but the actual experience isn't." d. "You can have an epidural. You won't feel anything."

ANS:B "It's normal to be anxious about labor. Let's discuss what makes you afraid" allows the woman to share her concerns with the nurse and is a therapeutic communication tool. "Don't worry about it. You'll do fine" negates the woman's fears and is not therapeutic. "Labor is scary to think about, but the actual experience isn't" negates the woman's fears and offers a false sense of security. It is not true that every woman may have an epidural. A number of criteria must be met for use of an epidural. Furthermore, many women still experience the feeling of pressure with an epidural.

A new client and her partner arrive on the labor, delivery, recovery, and postpartum unit for the birth of their first child. You apply the electronic fetal monitor (EFM) to the woman. Her partner asks you to explain what is printing on the graph, referring to the EFM strip. He wants to know what the baby's heart rate should be. Your best response is: a. "Don't worry about that machine; that's my job." b. "The top line graphs the baby's heart rate. Generally the heart rate is between 110 and 160. The heart rate will fluctuate in response to what is happening during labor." c. "The top line graphs the baby's heart rate, and the bottom line lets me know how strong the contractions are." d. "Your doctor will explain all of that later."

ANS:B "The top line graphs the baby's heart rate. Generally the heart rate is between 110 and 160. The heart rate will fluctuate in response to what is happening during labor" educates the partner about fetal monitoring and provides support and information to alleviate his fears. "Don't worry about that machine; that's my job" discredits the partner's feelings and does not provide the teaching he is requesting. "The top line graphs the baby's heart rate, and the bottom line lets me know how strong the contractions are" provides inaccurate information and does not address the partner's concerns about the fetal heart rate. The EFM graphs the frequency and duration of the contractions, not the intensity. Nurses should take every opportunity to provide client and family teaching, especially when information is requested.

A maternal indication for the use of vacuum extraction is: a. A wide pelvic outlet. c. A history of rapid deliveries. b. Maternal exhaustion. d. Failure to progress past 0 station.

ANS:B A mother who is exhausted may be unable to assist with the expulsion of the fetus. The patient with a wide pelvic outlet will likely not require vacuum extraction. With a rapid delivery, vacuum extraction is not necessary. A station of 0 is too high for a vacuum extraction.

The nerve block used in labor that provides anesthesia to the lower vagina and perineum is called: a. An epidural. c. A local. b. A pudendal. d. A spinal block.

ANS:B A pudendal block anesthetizes the lower vagina and perineum to provide anesthesia for an episiotomy and use of low forceps if needed. An epidural provides anesthesia for the uterus, perineum, and legs. A local provides anesthesia for the perineum at the site of the episiotomy. A spinal block provides anesthesia for the uterus, perineum, and down the legs.

Concerning the third stage of labor, nurses should be aware that: a. The placenta eventually detaches itself from a flaccid uterus. b. An expectant or active approach to managing this stage of labor reduces the risk of complications. c. It is important that the dark, roughened maternal surface of the placenta appear before the shiny fetal surface. d. The major risk for women during the third stage is a rapid heart rate.

ANS:B Active management facilitates placental separation and expulsion, thus reducing the risk of complications. The placenta cannot detach itself from a flaccid (relaxed) uterus. Which surface of the placenta comes out first is not clinically important. The major risk for women during the third stage of labor is postpartum hemorrhage.

A woman who is gravida 3 para 2 enters the intrapartum unit. The most important nursing assessments are: a. Contraction pattern, amount of discomfort, and pregnancy history. b. Fetal heart rate, maternal vital signs, and the woman's nearness to birth. c. Identification of ruptured membranes, the woman's gravida and para, and her support person. d. Last food intake, when labor began, and cultural practices the couple desires.

ANS:B All options describe relevant intrapartum nursing assessments; however, this focused assessment has priority. If the maternal and fetal conditions are normal and birth is not imminent, other assessments can be performed in an unhurried manner. This includes: gravida, para, support person, pregnancy history, pain assessment, last food intake, and cultural practices.

What is an expected characteristic of amniotic fluid? a. Deep yellow color b. Pale, straw color with small white particles c. Acidic result on a Nitrazine test d. Absence of ferning

ANS:B Amniotic fluid normally is a pale, straw-colored fluid that may contain white flecks of vernix. Yellow-stained fluid may indicate fetal hypoxia up to 36 hours before rupture of membranes, fetal hemolytic disease, or intrauterine infection. Amniotic fluid produces an alkaline result on a Nitrazine test. The presence of ferning is a positive indication of amniotic fluid.

Under which circumstance would it be unnecessary for the nurse to perform a vaginal examination? a. An admission to the hospital at the start of labor b. When accelerations of the fetal heart rate (FHR) are noted c. On maternal perception of perineal pressure or the urge to bear down d. When membranes rupture

ANS:B An accelerated FHR is a positive sign; however, variable decelerations merit a vaginal examination. Vaginal examinations should be performed when the woman is admitted, when she perceives perineal pressure or the urge to bear down, when her membranes rupture, when a significant change in her uterine activity has occurred, or when variable decelerations of the FHR are noted.

With regard to a pregnant woman's anxiety and pain experience, nurses should be aware that: a. Even mild anxiety must be treated. b. Severe anxiety increases tension, which increases pain, which in turn increases fear and anxiety, and so on. c. Anxiety may increase the perception of pain, but it does not affect the mechanism of labor. d. Women who have had a painful labor will have learned from the experience and have less anxiety the second time because of increased familiarity.

ANS:B Anxiety and pain reinforce each other in a negative cycle. Mild anxiety is normal for a woman in labor and likely needs no special treatment other than the standard reassurances. Anxiety increases muscle tension and ultimately can build sufficiently to slow the progress of labor. Unfortunately, an anxious, painful first labor is likely to carry over, through expectations and memories, into an anxious and painful experience in the second pregnancy.

After change-of-shift report the nurse assumes care of a multiparous client in labor. The woman is complaining of pain that radiates to her abdominal wall, lower back, and buttocks and down her thighs. Before implementing a plan of care, the nurse should understand that this type of pain is: a. Visceral. c. Somatic. b. Referred. d. Afterpain.

ANS:B As labor progresses the woman often experiences referred pain. This occurs when pain that originates in the uterus radiates to the abdominal wall, the lumbosacral area of the back, the gluteal area, and thighs. The woman usually has pain only during a contraction and is free from pain between contractions. Visceral pain is that which predominates in the first stage of labor. This pain originates from cervical changes, distention of the lower uterine segment, and uterine ischemia. Visceral pain is located over the lower portion of the abdomen. Somatic pain is described as intense, sharp, burning, and well localized. This results from stretching of the perineal tissues and the pelvic floor. This occurs during the second stage of labor. Pain experienced during the third stage of labor or afterward during the early postpartum period is uterine. This pain is very similar to that experienced in the first stage of labor.

While caring for the patient who requires an induction of labor, the nurse should be cognizant that: a. Ripening the cervix usually results in a decreased success rate for induction. b. Labor sometimes can be induced with balloon catheters or laminaria tents. c. Oxytocin is less expensive than prostaglandins and more effective but creates greater health risks. d. Amniotomy can be used to make the cervix more favorable for labor.

ANS:B Balloon catheters or laminaria tents are mechanical means of ripening the cervix. Ripening the cervix, making it softer and thinner, increases the success rate of induced labor. Prostaglandin E1 is less expensive and more effective than oxytocin but carries a greater risk. Amniotomy is the artificial rupture of membranes, which is used to induce labor only when the cervix is already ripe.

It is important for the nurse to develop a realistic birth plan with the pregnant woman in her care. The nurse can explain that a major advantage of nonpharmacologic pain management is: a. Greater and more complete pain relief is possible. b. No side effects or risks to the fetus are involved. c. The woman remains fully alert at all times. d. A more rapid labor is likely.

ANS:B Because nonpharmacologic pain management does not include analgesics, adjunct drugs, or anesthesia, it is harmless to the mother and the fetus. There is less pain relief with nonpharmacologic pain management during childbirth. The woman's alertness is not altered by medication; however, the increase in pain will decrease alertness. Pain management may or may not alter the length of labor. At times when pain is decreased, the mother relaxes and labor progresses at a quicker pace.

Nurses should know some basic definitions concerning preterm birth, preterm labor, and low birth weight. For instance: a. The terms preterm birth and low birth weight can be used interchangeably. b. Preterm labor is defined as cervical changes and uterine contractions occurring between 20 and 37 weeks of pregnancy. c. Low birth weight is anything below 3.7 pounds. d. In the United States early in this century, preterm birth accounted for 18% to 20% of all births.

ANS:B Before 20 weeks, it is not viable (miscarriage); after 37 weeks, it can be considered term. Although these terms are used interchangeably, they have different meanings: preterm birth describes the length of gestation (37 weeks) regardless of weight; low birth weight describes weight only (2500 g or less) at the time of birth, whenever it occurs. Low birth weight is anything less than 2500 g, or about 5.5 pounds. In 2003 the preterm birth rate in the United States was 12.3%, but it is increasing in frequency.

Which statement is the best rationale for assessing maternal vital signs between contractions? a. During a contraction, assessing fetal heart rates is the priority. b. Maternal circulating blood volume increases temporarily during contractions. c. Maternal blood flow to the heart is reduced during contractions. d. Vital signs taken during contractions are not accurate.

ANS:B During uterine contractions, blood flow to the placenta temporarily stops, causing a relative increase in the mother's blood volume, which in turn temporarily increases blood pressure and slows pulse. It is important to monitor fetal response to contractions; however, this question is concerned with the maternal vital signs. Maternal blood flow is increased during a contraction. Vital signs are altered by contractions but are considered accurate for that period of time.

With regard to systemic analgesics administered during labor, nurses should be aware that: a. Systemic analgesics cross the maternal blood-brain barrier as easily as they do the fetal blood-brain barrier. b. Effects on the fetus and newborn can include decreased alertness and delayed sucking. c. Intramuscular administration (IM) is preferred over intravenous (IV) administration. d. IV patient-controlled analgesia (PCA) results in increased use of an analgesic.

ANS:B Effects depend on the specific drug given, the dosage, and the timing. Systemic analgesics cross the fetal blood-brain barrier more readily than the maternal blood-brain barrier. IV administration is preferred over IM administration because the drug acts faster and more predictably. PCA results in decreased use of an analgesic.

The slight overlapping of cranial bones or shaping of the fetal head during labor is called: a. Lightening. c. Ferguson reflex. b. Molding. d. Valsalva maneuver.

ANS:B Fetal head formation is called molding. Molding also permits adaptation to various diameters of the maternal pelvis. Lightening is the mother's sensation of decreased abdominal distention, which usually occurs the week before labor. The Ferguson reflex is the contraction urge of the uterus after stimulation of the cervix. The Valsalva maneuver describes conscious pushing during the second stage of labor.

With regard to spinal and epidural (block) anesthesia, nurses should know that: a. This type of anesthesia is commonly used for cesarean births but is not suitable for vaginal births. b. A high incidence of after-birth headache is seen with spinal blocks. c. Epidural blocks allow the woman to move freely. d. Spinal and epidural blocks are never used together.

ANS:B Headaches may be prevented or mitigated to some degree by a number of methods. Spinal blocks may be used for vaginal births, but the woman must be assisted through labor. Epidural blocks limit the woman's ability to move freely. Combined use of spinal and epidural blocks is becoming increasingly popular.

The primary difference between the labor of a nullipara and that of a multipara is the: a. Amount of cervical dilation. c. Level of pain experienced. b. Total duration of labor. d. Sequence of labor mechanisms.

ANS:B In a first-time pregnancy, descent is usually slow but steady; in subsequent pregnancies, descent is more rapid, resulting in a shorter duration of labor. Cervical dilation is the same for all labors. Level of pain is individual to the woman, not to the number of labors she has experienced. The sequence of labor mechanisms is the same with all labors.

The primary difference between the labor of a nullipara and that of a multipara is the: a. Amount of cervical dilation. c. Level of pain experienced. b. Total duration of labor. d. Sequence of labor mechanisms.

ANS:B Multiparas usually labor more quickly than nulliparas, thus making the total duration of their labor shorter. Cervical dilation is the same for all labors. The level of pain is individual to the woman, not to the number of labors she has experienced. The sequence of labor mechanisms remains the same with all labors.

The role of the nurse with regard to informed consent is to: a. Inform the client about the procedure and have her sign the consent form. b. Act as a client advocate and help clarify the procedure and the options. c. Call the physician to see the client. d. Witness the signing of the consent form.

ANS:B Nurses play a part in the informed consent process by clarifying and describing procedures or by acting as the woman's advocate and asking the primary health care provider for further explanations. The physician is responsible for informing the woman of her options, explaining the procedure, and advising the client about potential risk factors. The physician must be present to explain the procedure to the client. However, the nurse's responsibilities go further than simply asking the physician to see the client. The nurse may witness the signing of the consent form. However, depending on the state's guidelines, the woman's husband or another hospital health care employee may sign as witness.

The nurse expects to administer an oxytocic (e.g., Pitocin, Methergine) to a woman after expulsion of her placenta to: a. Relieve pain. c. Prevent infection. b. Stimulate uterine contraction. d. Facilitate rest and relaxation.

ANS:B Oxytocics stimulate uterine contractions, which reduce blood loss after the third stage of labor. Oxytocics are not used to treat pain or prevent infection. They cause the uterus to contract, which reduces blood loss. Oxytocics do not facilitate rest and relaxation.

With regard to dysfunctional labor, nurses should be aware that: a. Women who are underweight are more at risk. b. Women experiencing precipitous labor are about the only "dysfunctionals" not to be exhausted. c. Hypertonic uterine dysfunction is more common than hypotonic dysfunction. d. Abnormal labor patterns are most common in older women.

ANS:B Precipitous labor lasts less than 3 hours. Short women more than 30 pounds overweight are more at risk for dysfunctional labor. Hypotonic uterine dysfunction, in which the contractions become weaker, is more common. Abnormal labor patterns are more common in women less than 20 years of age.

What three measures should the nurse implement to provide intrauterine resuscitation? Select the response that best indicates the priority of actions that should be taken. a. Call the provider, reposition the mother, and perform a vaginal examination. b. Reposition the mother, increase intravenous (IV) fluid, and provide oxygen via face mask. c. Administer oxygen to the mother, increase IV fluid, and notify the care provider. d. Perform a vaginal examination, reposition the mother, and provide oxygen via face mask.

ANS:B Repositioning the mother, increasing intravenous (IV) fluid, and providing oxygen via face mask are correct nursing actions for intrauterine resuscitation. The nurse should initiate intrauterine resuscitation in an ABC manner, similar to basic life support. The first priority is to open the maternal and fetal vascular systems by repositioning the mother for improved perfusion. The second priority is to increase blood volume by increasing the IV fluid. The third priority is to optimize oxygenation of the circulatory volume by providing oxygen via face mask. If these interventions do not resolve the fetal heart rate issue quickly, the primary provider should be notified immediately.

A first-time mother is concerned about the type of medications she will receive during labor. She is in a fair amount of pain and is nauseous. In addition, she appears to be very anxious. You explain that opioid analgesics often are used with sedatives because: a. "The two together work the best for you and your baby." b. "Sedatives help the opioid work better, and they also will assist you to relax and relieve your nausea." c. "They work better together so you can sleep until you have the baby." d. "This is what the doctor has ordered for you."

ANS:B Sedatives can be used to reduce the nausea and vomiting that often accompany opioid use. In addition, some ataractics reduce anxiety and apprehension and potentiate the opioid analgesic affects. A potentiator may cause the two drugs to work together more effectively, but it does not ensure maternal or fetal complications will not occur. Sedation may be a related effect of some ataractics, but it is not the goal. Furthermore, a woman is unlikely to be able to sleep through transitional labor and birth. "This is what the doctor has ordered for you" may be true, but it is not an acceptable comment for the nurse to make.

After an emergency birth, the nurse encourages the woman to breastfeed her newborn. The primary purpose of this activity is to: a. Facilitate maternal-newborn interaction. b. Stimulate the uterus to contract. c. Prevent neonatal hypoglycemia. d. Initiate the lactation cycle.

ANS:B Stimulation of the nipples through breastfeeding or manual stimulation causes the release of oxytocin and prevents maternal hemorrhage. Breastfeeding facilitates maternal-newborn interaction, but it is not the primary reason a woman is encouraged to breastfeed after an emergency birth. The primary intervention for preventing neonatal hypoglycemia is thermoregulation. Cold stress can result in hypoglycemia. The woman is encouraged to breastfeed after an emergency birth to stimulate the release of oxytocin, which prevents hemorrhage. Breastfeeding is encouraged to initiate the lactation cycle, but it is not the primary reason for this activity after an emergency birth.

The nurse providing care for a woman with preterm labor who is receiving terbutaline would include which intervention to identify side effects of the drug? a. Assessing deep tendon reflexes (DTRs) b. Assessing for chest discomfort and palpitations c. Assessing for bradycardia d. Assessing for hypoglycemia

ANS:B Terbutaline is a b2-adrenergic agonist that affects the cardiopulmonary and metabolic systems of the mother. Signs of cardiopulmonary decompensation would include chest pain and palpitations. Assessing DTRs would not address these concerns. b2-Adrenergic agonist drugs cause tachycardia, not bradycardia. The metabolic effect leads to hyperglycemia, not hypoglycemia.

Nurses can help their clients by keeping them informed about the distinctive stages of labor. What description of the phases of the first stage of labor is accurate? a. Latent: Mild, regular contractions; no dilation; bloody show; duration of 2 to 4 hours b. Active: Moderate, regular contractions; 4- to 7-cm dilation; duration of 3 to 6 hours c. Lull: No contractions; dilation stable; duration of 20 to 60 minutes d. Transition: Very strong but irregular contractions; 8- to 10-cm dilation; duration of 1 to 2 hours

ANS:B The active phase is characterized by moderate, regular contractions; 4- to 7-cm dilation; and a duration of 3 to 6 hours. The latent phase is characterized by mild-to-moderate, irregular contractions; dilation up to 3 cm; brownish-to-pale pink mucus, and a duration of 6 to 8 hours. No official "lull" phase exists in the first stage. The transition phase is characterized by strong to very strong, regular contractions; 8- to 10-cm dilation; and a duration of 20 to 40 minutes.

The nurse should be aware that an effective plan to achieve adequate pain relief without maternal risk is most effective if: a. The mother gives birth without any analgesic or anesthetic. b. The mother and family's priorities and preferences are incorporated into the plan. c. The primary health care provider decides the best pain relief for the mother and family. d. The nurse informs the family of all alternative methods of pain relief available in the hospital setting.

ANS:B The assessment of the woman, her fetus, and her labor is a joint effort of the nurse and the primary health care providers, who consult with the woman about their findings and recommendations. The needs of each woman are different, and many factors must be considered before a decision is made whether pharmacologic methods, nonpharmacologic methods, or a combination of the two will be used to manage labor pain.

During labor a fetus with an average heart rate of 135 beats/min over a 10-minute period would be considered to have: a. Bradycardia. c. Tachycardia. b. A normal baseline heart rate. d. Hypoxia.

ANS:B The baseline heart rate is measured over 10 minutes; a normal range is 110 to 160 beats/min. Bradycardia is a fetal heart rate (FHR) below 110 beats/min for 10 minutes or longer. Tachycardia is an FHR over 160 beats/min for 10 minutes or longer. Hypoxia is an inadequate supply of oxygen; no indication of this condition exists with a baseline heart rate in the normal range.

A pregnant woman at 29 weeks of gestation has been diagnosed with preterm labor. Her labor is being controlled with tocolytic medications. She asks when she would be able to go home. Which response by the nurse is most accurate? a. "After the baby is born." b. "When we can stabilize your preterm labor and arrange home health visits." c. "Whenever the doctor says that it is okay." d. "It depends on what kind of insurance coverage you have."

ANS:B The client's preterm labor is being controlled with tocolytics. Once she is stable, home care may be a viable option for this type of client. Care of a woman with preterm labor is multifactorial; the goal is to prevent delivery. In many cases this may be achieved at home. Care of the preterm client is multidisciplinary and multifactorial. Managed care may dictate earlier hospital discharges or a shift from hospital to home care. Insurance coverage may be one factor in the care of clients, but ultimately client safety remains the most important factor.

To teach patients about the process of labor adequately, the nurse knows that which event is the best indicator of true labor? a. Bloody show c. Fetal descent into the pelvic inlet b. Cervical dilation and effacement d. Uterine contractions every 7 minutes

ANS:B The conclusive distinction between true and false labor is that contractions of true labor cause progressive change in the cervix. Bloody show can occur before true labor. Fetal descent can occur before true labor. False labor may have contractions that occur this frequently; however, this is usually inconsistent.

The nurse has received report regarding her patient in labor. The woman's last vaginal examination was recorded as 3 cm, 30%, and ?2-2. The nurse's interpretation of this assessment is that: a. The cervix is effaced 3 cm, it is dilated 30%, and the presenting part is 2 cm above the ischial spines. b. The cervix is 3 cm dilated, it is effaced 30%, and the presenting part is 2 cm above the ischial spines. c. The cervix is effaced 3 cm, it is dilated 30%, and the presenting part is 2 cm below the ischial spines. d. The cervix is dilated 3 cm, it is effaced 30%, and the presenting part is 2 cm below the ischial spines.

ANS:B The correct description of the vaginal examination for this woman in labor is the cervix is 3 cm dilated, it is effaced 30%, and the presenting part is 2 cm above the ischial spines. The sterile vaginal examination is recorded as centimeters of cervical dilation, percentage of cervical dilation, and the relationship of the presenting part to the ischial spines (either above or below).

Through vaginal examination the nurse determines that a woman is 4 cm dilated, and the external fetal monitor shows uterine contractions every 3.5 to 4 minutes. The nurse would report this as: a. First stage, latent phase. c. First stage, transition phase. b. First stage, active phase. d. Second stage, latent phase.

ANS:B The first stage, active phase of maternal progress indicates that the woman is in the active phase of the first stage of labor. During the latent phase of the first stage of labor, the expected maternal progress would be 0 to 3 cm dilation with contractions every 5 to 30 minutes. During the transition phase of the first stage of labor, the expected maternal progress is 8 to 10 cm dilation with contractions every 2 to 3 minutes. During the latent phase of the second stage of labor, the woman is completely dilated and experiences a restful period of "laboring down."

A patient whose cervix is dilated to 5 cm is considered to be in which phase of labor? a. Latent phase c. Second stage b. Active phase d. Third stage

ANS:B The latent phase is from the beginning of true labor until 3 cm of cervical dilation. The active phase of labor is characterized by cervical dilation of 4 to 7 cm. The second stage of labor begins when the cervix is completely dilated until the birth of the baby. The third stage of labor is from the birth of the baby until the expulsion of the placenta. This patient is in the active phase of labor.

For women who have a history of sexual abuse, a number of traumatic memories may be triggered during labor. The woman may fight the labor process and react with pain or anger. Alternately, she may become a passive player and emotionally absent herself from the process. The nurse is in a unique position of being able to assist the client to associate the sensations of labor with the process of childbirth and not the past abuse. The nurse can implement a number of care measures to help the client view the childbirth experience in a positive manner. Which intervention would be key for the nurse to use while providing care? a. Telling the client to relax and that it won't hurt much b. Limiting the number of procedures that invade her body c. Reassuring the client that as the nurse you know what is best d. Allowing unlimited care providers to be with the client

ANS:B The number of invasive procedures such as vaginal examinations, internal monitoring, and intravenous therapy should be limited as much as possible. The nurse should always avoid words and phrases that may result in the client's recalling the phrases of her abuser (e.g., "Relax, this won't hurt" or "Just open your legs.") The woman's sense of control should be maintained at all times. The nurse should explain procedures at the client's pace and wait for permission to proceed. Protecting the client's environment by providing privacy and limiting the number of staff who observe the client will help to make her feel safe.

Nurses alert to signs of the onset of the second stage of labor can be certain that this stage has begun when: a. The woman has a sudden episode of vomiting. b. The nurse is unable to feel the cervix during a vaginal examination. c. Bloody show increases. d. The woman involuntarily bears down.

ANS:B The only certain objective sign that the second stage has begun is the inability to feel the cervix because it is fully dilated and effaced. Vomiting, an increase in bloody show, and involuntary bearing down are only suggestions of second-stage labor.

As relates to fetal positioning during labor, nurses should be aware that: a. Position is a measure of the degree of descent of the presenting part of the fetus through the birth canal. b. Birth is imminent when the presenting part is at +4 to +5 cm below the spine. c. The largest transverse diameter of the presenting part is the suboccipitobregmatic diameter. d. Engagement is the term used to describe the beginning of labor.

ANS:B The station of the presenting part should be noted at the beginning of labor so that the rate of descent can be determined. Position is the relation of the presenting part of the fetus to the four quadrants of the mother's pelvis;station is the measure of degree of descent. The largest diameter usually is the biparietal diameter. The suboccipitobregmatic diameter is the smallest, although one of the most critical. Engagement often occurs in the weeks just before labor in nulliparas and before or during labor in multiparas.

To assist the woman after delivery of the infant, the nurse knows that the blood patch is used after spinal anesthesia to relieve: a. Hypotension. c. Neonatal respiratory depression. b. Headache. d. Loss of movement.

ANS:B The subarachnoid block may cause a postspinal headache resulting from loss of cerebrospinal fluid from the puncture in the dura. When blood is injected into the epidural space in the area of the dural puncture, it forms a seal over the hole to stop leaking of cerebrospinal fluid. Hypotension is prevented by increasing fluid volume before the procedure. Neonatal respiratory depression is not an expected outcome with spinal anesthesia. Loss of movement is an expected outcome of spinal anesthesia.

Why is continuous electronic fetal monitoring usually used when oxytocin is administered? a. The mother may become hypotensive. b. Uteroplacental exchange may be compromised. c. Maternal fluid volume deficit may occur. d. Fetal chemoreceptors are stimulated.

ANS:B The uterus may contract more firmly, and the resting tone may be increased with oxytocin use. This response reduces entrance of freshly oxygenated maternal blood into the intervillous spaces, thus depleting fetal oxygen reserves. Hypotension is not a common side effect of oxytocin. All laboring women are at risk for fluid volume deficit; oxytocin administration does not increase the risk. Oxytocin affects the uterine muscles.

If an opioid antagonist is administered to a laboring woman, she should be told that: a. Her pain will decrease. b. Her pain will return. c. She will feel less anxious. d. She will no longer feel the urge to push.

ANS:B The woman should be told that the pain that was relieved by the opioid analgesic will return with administration of the opioid antagonist. Opioid antagonists, such as Narcan, promptly reverse the central nervous system (CNS) depressant effects of opioids. In addition, the antagonist counters the effect of the stress-induced levels of endorphins. An opioid antagonist is especially valuable if labor is more rapid than expected and birth is anticipated when the opioid is at its peak effect.

The nurse is caring for a client whose labor is being augmented with oxytocin. He or she recognizes that the oxytocin should be discontinued immediately if there is evidence of: a. Uterine contractions occurring every 8 to 10 minutes. b. A fetal heart rate (FHR) of 180 with absence of variability. c. The client's needing to void. d. Rupture of the client's amniotic membranes.

ANS:B This FHR is nonreassuring. The oxytocin should be discontinued immediately, and the physician should be notified. The oxytocin should be discontinued if uterine hyperstimulation occurs. Uterine contractions that are occurring every 8 to 10 minutes do not qualify as hyperstimulation. The client's needing to void is not an indication to discontinue the oxytocin induction immediately or to call the physician. Unless a change occurs in the FHR pattern that is nonreassuring or the client experiences uterine hyperstimulation, the oxytocin does not need to be discontinued. The physician should be notified that the client's membranes have ruptured.

The nurse providing care for the laboring woman realizes that variable fetal heart rate (FHR) decelerations are caused by: a. Altered fetal cerebral blood flow. c. Uteroplacental insufficiency. b. Umbilical cord compression. d. Fetal hypoxemia.

ANS:B Variable decelerations can occur any time during the uterine contracting phase and are caused by compression of the umbilical cord. Altered fetal cerebral blood flow would result in early decelerations in the FHR. Uteroplacental insufficiency would result in late decelerations in the FHR. Fetal hypoxemia would result in tachycardia initially and then bradycardia if hypoxia continues.

A female infertility patient is found to be hypoestrogenic at the preconceptual clinic visit. She asks the nurse why she has never been able to get pregnant. Which response is best? a. Circulating estrogen contributes to secondary sex characteristics. b. Estrogen deficiency prevents the ovum from reaching the uterus and may be a factor in infertility. c. Hyperestrogen may be preventing the zona pellucida from forming an ovum protective layer. d. The corona radiata is preventing fertilization of the ovum.

ANS:B, The cilia in the tubes are stimulated by high estrogen levels, which propel the ovum toward the uterus. Without estrogen, the ovum won't reach the uterus. The results of a series of events occurring in the ovary cause an expulsion of the oocyte from the ovarian follicle known as ovulation. The ovarian cycle is driven by multiple important hormones: 1) gonadotropic hormone, 2) follicle stimulating hormone (FSH), and 3) luteinizing hormone (LH). The cilia in the tubes are stimulated by high 4) estrogen levels, which propel the ovum toward the uterus. The zona pellucida (inner layer) and corona radiata (outer layer) form protective layers around the ovum. If an ovum is not fertilized within 24 hours of ovulation by a sperm, it is usually reabsorbed into a woman's body. A patient who is hypoestrogenic would not have excess circulating estrogen. A patient with low estrogen would not be classified as hyperestrogenic. Without sufficient estrogen, there can be no fertilization of the ovum.

Maternal hypotension is a potential side effect of regional anesthesia and analgesia. What nursing interventions could you use to raise the client's blood pressure (Select all that apply)? a. Place the woman in a supine position. b. Place the woman in a lateral position. c. Increase intravenous (IV) fluids. d. Administer oxygen. e. Perform a vaginal examination.

ANS:B,C,D Nursing interventions for maternal hypotension arising from analgesia or anesthesia include turning the woman to a lateral position, increasing IV fluids, administering oxygen via face mask, elevating the woman's legs, notifying the physician, administering an IV vasopressor, and monitoring the maternal and fetal status at least every 5 minutes until these are stable. Placing the client in a supine position would cause venous compression, thereby limiting blood flow to and oxygenation of the placenta and fetus. A sterile vaginal examination has no bearing on maternal blood pressure.

Complications and risks associated with cesarean births include (Select all that apply): a. Placental abruption. b. Wound dehiscence. c. Hemorrhage. d. Urinary tract infections. e. Fetal injuries.

ANS:B,C,D,E, Placental abruption and placenta previa are both indications for cesarean birth and are not complications thereof. Wound dehiscence, hemorrhage, urinary tract infection, and fetal injuries are all possible complications and risks associated with delivery by cesarean section.

A tiered system of categorizing FHR has been recommended by regulatory agencies. Nurses, midwives, and physicians who care for women in labor must have a working knowledge of fetal monitoring standards and understand the significance of each category. These categories include (Select all that apply): a. Reassuring. b. Category I. c. Category II. d. Nonreassuring. e. Category III.

ANS:B,C,E The three tiered system of FHR tracings include Category I, II, and III. Category I is a normal tracing requiring no action. Category II FHR tracings are indeterminate. This category includes tracings that do not meet Category I or III criteria. Category III tracings are abnormal and require immediate intervention.

The nurse recognizes that uterine hyperstimulation with oxytocin requires emergency interventions. What clinical cues would alert the nurse that the woman is experiencing uterine hyperstimulation (Select all that apply)? a. Uterine contractions lasting <90 seconds and occurring >2 minutes in frequency b. Uterine contractions lasting >90 seconds and occurring <2 minutes in frequency c. Uterine tone <20 mm Hg d. Uterine tone >20 mm Hg e. Increased uterine activity accompanied by a nonreassuring fetal heart rate (FHR) and pattern

ANS:B,D,E Uterine contractions that occur less than 2 minutes apart and last more than 90 seconds, a uterine tone of over 20 mm Hg, and a nonreassuring FHR and pattern are all indications of uterine hyperstimulation with oxytocin administration. Uterine contractions that occur more than 2 minutes apart and last less than 90 seconds are the expected goal of oxytocin induction. A uterine tone of less than 20 mm Hg is normal.

Which nursing assessment indicates that a woman who is in second-stage labor is almost ready to give birth? a. The fetal head is felt at 0 station during vaginal examination. b. Bloody mucus discharge increases. c. The vulva bulges and encircles the fetal head. d. The membranes rupture during a contraction.

ANS:C A bulging vulva that encircles the fetal head describes crowning, which occurs shortly before birth. Birth of the head occurs when the station is +4. A 0 station indicates engagement. Bloody show occurs throughout the labor process and is not an indication of an imminent birth. Rupture of membranes can occur at any time during the labor process and does not indicate an imminent birth.

It is paramount for the obstetric nurse to understand the regulatory procedures and criteria for admitting a woman to the hospital labor unit. Which guideline is an important legal requirement of maternity care? a. The patient is not considered to be in true labor (according to the Emergency Medical Treatment and Active Labor Act [EMTALA]) until a qualified health care provider says she is. b. The woman can have only her male partner or predesignated "doula" with her at assessment. c. The patient's weight gain is calculated to determine whether she is at greater risk for cephalopelvic disproportion (CPD) and cesarean birth. d. The nurse may exchange information about the patient with family members.

ANS:C According to EMTALA, a woman is entitled to active labor care and is presumed to be in "true" labor until a qualified health care provider certifies otherwise. A woman can have anyone she wishes present for her support. The risk for CPD is especially great for petite women or those who have gained 16 kg or more. All patients should have their weight and BMI calculated on admission. This is part of standard nursing care on a maternity unit and not a regulatory concern. According to the Health Insurance Portability and Accountability Act (HIPAA), the patient must give consent for others to receive any information related to her condition.

A laboring woman received an opioid agonist (meperidine) intravenously 90 minutes before she gave birth. Which medication should be available to reduce the postnatal effects of Demerol on the neonate? a. Fentanyl (Sublimaze) c. Naloxone (Narcan) b. Promethazine (Phenergan) d. Nalbuphine (Nubain)

ANS:C An opioid antagonist can be given to the newborn as one part of the treatment for neonatal narcosis, which is a state of central nervous system (CNS) depression in the newborn produced by an opioid. Opioid antagonists such as naloxone (Narcan) can promptly reverse the CNS depressant effects, especially respiratory depression. Fentanyl, promethazine, and nalbuphine do not act as opioid antagonists to reduce the postnatal effects of Demerol on the neonate. Although meperidine (Demerol) is a low-cost medication and readily available, the use of Demerol in labor has been controversial because of its effects on the neonate.

When assessing a woman in labor, the nurse is aware that the relationship of the fetal body parts to one another is called fetal: a.Lie. c.Attitude. b.Presentation. d.Position.

ANS:C Attitude is the relation of the fetal body parts to one another. Lie is the relation of the long axis (spine) of the fetus to the long axis (spine) of the mother. Presentation refers to the part of the fetus that enters the pelvic inlet first and leads through the birth canal during labor at term. Position is the relation of the presenting part to the four quadrants of the mother's pelvis.

The priority nursing care associated with an oxytocin (Pitocin) infusion is: a. Measuring urinary output. b. Increasing infusion rate every 30 minutes. c. Monitoring uterine response. d. Evaluating cervical dilation.

ANS:C Because of the risk of hyperstimulation, which could result in decreased placental perfusion and uterine rupture, the nurse's priority intervention is monitoring uterine response. Monitoring urinary output is also important; however, it is not the top priority during the administration of Pitocin. The infusion rate may be increased after proper assessment that it is an appropriate interval to do so. Monitoring labor progression is the standard of care for all labor patients.

As relates to the use of tocolytic therapy to suppress uterine activity, nurses should be aware that: a. The drugs can be given efficaciously up to the designated beginning of term at 37 weeks. b. There are no important maternal (as opposed to fetal) contraindications. c. Its most important function is to afford the opportunity to administer antenatal glucocorticoids. d. If the client develops pulmonary edema while receiving tocolytics, intravenous (IV) fluids should be given.

ANS:C Buying time for antenatal glucocorticoids to accelerate fetal lung development may be the best reason to use tocolytics. Once the pregnancy has reached 34 weeks, the risks of tocolytic therapy outweigh the benefits. There are important maternal contraindications to tocolytic therapy. Tocolytic-induced edema can be caused by IV fluids.

A nulliparous woman who has just begun the second stage of her labor would most likely: a. Experience a strong urge to bear down. b. Show perineal bulging. c. Feel tired yet relieved that the worst is over. d. Show an increase in bright red bloody show.

ANS:C Common maternal behaviors during the latent phase of the second stage of labor include feeling a sense of accomplishment and optimism because "the worst is over." During the latent phase of the second stage of labor, the urge to bear down often is absent or only slight during the acme of contractions. Perineal bulging occurs during the transition phase of the second stage of labor, not at the beginning of the second stage. An increase in bright red bloody show occurs during the descent phase of the second stage of labor.

Which nursing assessment indicates that a woman who is in second-stage labor is almost ready to give birth? a. The fetal head is felt at 0 station during vaginal examination. b. Bloody mucus discharge increases. c. The vulva bulges and encircles the fetal head. d. The membranes rupture during a contraction.

ANS:C During the active pushing (descent) phase, the woman has strong urges to bear down as the presenting part of the fetus descends and presses on the stretch receptors of the pelvic floor. The vulva stretches and begins to bulge encircling the fetal head. Birth of the head occurs when the station is +4. A 0 station indicates engagement. Bloody show occurs throughout the labor process and is not an indication of an imminent birth. Rupture of membranes can occur at any time during the labor process and does not indicate an imminent birth.

The nurse knows that the second stage of labor, the descent phase, has begun when: a. The amniotic membranes rupture. b. The cervix cannot be felt during a vaginal examination. c. The woman experiences a strong urge to bear down. d. The presenting part is below the ischial spines.

ANS:C During the descent phase of the second stage of labor, the woman may experience an increase in the urge to bear down. Rupture of membranes has no significance in determining the stage of labor. The second stage of labor begins with full cervical dilation. Many women may have an urge to bear down when the presenting part is below the level of the ischial spines. This can occur during the first stage of labor, as early as 5-cm dilation.

Fetal bradycardia is most common during: a. Intraamniotic infection. b. Fetal anemia. c. Prolonged umbilical cord compression. d. Tocolytic treatment using terbutaline.

ANS:C Fetal bradycardia can be considered a later sign of fetal hypoxia and is known to occur before fetal death. Bradycardia can result from placental transfer of drugs, prolonged compression of the umbilical cord, maternal hypothermia, and maternal hypotension. Intraamniotic infection, fetal anemia, and tocolytic treatment using terbutaline would most likely result in fetal tachycardia.

While providing care to a patient in active labor, the nurse should instruct the woman that: a. The supine position commonly used in the United States increases blood flow. b. The "all fours" position, on her hands and knees, is hard on her back. c. Frequent changes in position will help relieve her fatigue and increase her comfort. d. In a sitting or squatting position, her abdominal muscles will have to work harder.

ANS:C Frequent position changes relieve fatigue, increase comfort, and improve circulation. Blood flow can be compromised in the supine position; any upright position benefits cardiac output. The "all fours" position is used to relieve backache in certain situations. In a sitting or squatting position, the abdominal muscles work in greater harmony with uterine contractions.

Nurses with an understanding of cultural differences regarding likely reactions to pain may be better able to help clients. Nurses should know that _____ women may be stoic until late in labor, when they may become vocal and request pain relief. a. Chinese c. Hispanic b. Arab or Middle Eastern d. African-American

ANS:C Hispanic women may be stoic early and more vocal and ready for medications later. Chinese women may not show reactions to pain. Medical interventions must be offered more than once. Arab or Middle Eastern women may be vocal in response to labor pain from the start. They may prefer pain medications. African-American women may express pain openly; use of medications for pain is more likely to vary with the individual.

The laboring woman who imagines her body opening to let the baby out is using a mental technique called: a. Dissociation. c. Imagery. b. Effleurage. d. Distraction.

ANS:C Imagery is a technique of visualizing images that will assist the woman in coping with labor. Dissociation helps the woman learn to relax all muscles except those that are working. Effleurage is self-massage. Distraction can be used in the early latent phase by having the woman engage in another activity.

The nurse thoroughly dries the infant immediately after birth primarily to: a. Stimulate crying and lung expansion. b. Remove maternal blood from the skin surface. c. Reduce heat loss from evaporation. d. Increase blood supply to the hands and feet.

ANS:C Infants are wet with amniotic fluid and blood at birth, and this accelerates evaporative heat loss. The primary purpose of drying the infant is to prevent heat loss. Rubbing the infant does stimulate crying; however, it is not the main reason for drying the infant. This process does not remove all the maternal blood.

Prepidil (prostaglandin gel) has been ordered for a pregnant woman at 43 weeks of gestation. The nurse recognizes that this medication will be administered to: a. Enhance uteroplacental perfusion in an aging placenta. b. Increase amniotic fluid volume. c. Ripen the cervix in preparation for labor induction. d. Stimulate the amniotic membranes to rupture.

ANS:C It is accurate to state that Prepidil will be administered to ripen the cervix in preparation for labor induction. It is not administered to enhance uteroplacental perfusion in an aging placenta, increase amniotic fluid volume, or stimulate the amniotic membranes to rupture.

The nurse caring for the woman in labor should understand that maternal hypotension can result in: a. Early decelerations. c. Uteroplacental insufficiency. b. Fetal dysrhythmias. d. Spontaneous rupture of membranes.

ANS:C Low maternal blood pressure reduces placental blood flow during uterine contractions and results in fetal hypoxemia. Maternal hypotension is not associated with early decelerations, fetal dysrhythmias, or spontaneous rupture of membranes.

The nurse caring for the woman in labor should understand that increased variability of the fetal heart rate may be caused by: a. Narcotics. c. Methamphetamines. b. Barbiturates. d. Tranquilizers.

ANS:C Narcotics, barbiturates, and tranquilizers may be causes of decreased variability; methamphetamines may cause increased variability.

Because the risk for childbirth complications may be revealed, nurses should know that the point of maximal intensity (PMI) of the fetal heart tone (FHT) is: a. Usually directly over the fetal abdomen. b. In a vertex position heard above the mother's umbilicus. c. Heard lower and closer to the midline of the mother's abdomen as the fetus descends and rotates internally. d. In a breech position heard below the mother's umbilicus.

ANS:C Nurses should be prepared for the shift. The PMI of the FHT usually is directly over the fetal back. In a vertex position it is heard below the mother's umbilicus. In a breech position it is heard above the mother's umbilicus.

Surgical, medical, or mechanical methods may be used for labor induction. Which technique is considered a mechanical method of induction? a. Amniotomy c. Transcervical catheter b. Intravenous Pitocin d. Vaginal insertion of prostaglandins

ANS:C Placement of a balloon-tipped Foley catheter into the cervix is a mechanical method of induction. Other methods to expand and gradually dilate the cervix include hydroscopic dilators such as laminaria tents (made from desiccated seaweed), or Lamicel (contains magnesium sulfate). Amniotomy is a surgical method of augmentation and induction. Intravenous Pitocin and insertion of prostaglandins are medical methods of induction.

In planning for home care of a woman with preterm labor, which concern must the nurse address? a. Nursing assessments will be different from those done in the hospital setting. b. Restricted activity and medications will be necessary to prevent recurrence of preterm labor. c. Prolonged bed rest may cause negative physiologic effects. d. Home health care providers will be necessary.

ANS:C Prolonged bed rest may cause adverse effects such as weight loss, loss of appetite, muscle wasting, weakness, bone demineralization, decreased cardiac output, risk for thrombophlebitis, alteration in bowel functions, sleep disturbance, and prolonged postpartum recovery. Nursing assessments will differ somewhat from those performed in the acute care setting, but this is not the concern that needs to be addressed. Restricted activity and medication may prevent preterm labor, but not in all women. In addition, the plan of care is individualized to meet the needs of each woman. Many women will receive home health nurse visits, but care is individualized for each woman.

The nurse caring for a woman in labor understands that prolonged decelerations: a. Are a continuing pattern of benign decelerations that do not require intervention. b. Constitute a baseline change when they last longer than 5 minutes. c. Usually are isolated events that end spontaneously. d. Require the usual fetal monitoring by the nurse.

ANS:C Prolonged decelerations usually are isolated events that end spontaneously. However, in certain combinations with late and/or variable decelerations, they are a danger sign that requires the nurse to notify the physician or midwife immediately. A deceleration that lasts longer than 10 minutes constitutes a baseline change.

The nurse recognizes that a woman is in true labor when she states: a. "I passed some thick, pink mucus when I urinated this morning." b. "My bag of waters just broke." c. "The contractions in my uterus are getting stronger and closer together." d. "My baby dropped, and I have to urinate more frequently now."

ANS:C Regular, strong contractions with the presence of cervical change indicate that the woman is experiencing true labor. Loss of the mucous plug (operculum) often occurs during the first stage of labor or before the onset of labor, but it is not the indicator of true labor. Spontaneous rupture of membranes often occurs during the first stage of labor, but it is not the indicator of true labor. The presenting part of the fetus typically becomes engaged in the pelvis at the onset of labor, but this is not the indicator of true labor.

Many females experience problems achieving and maintaining a pregnancy. The ER nursing assessment of a child-bearing-age female shows back pain, elevated blood pressure, and leaking of clear fluid from the vagina. Maternal-fetal complications described above are most often associated with which child-bearing stage? a. Preconception b. First trimester c.Second-third trimester d.Postpartum

ANS:C Second and third trimester complications include anencephalus, chromosomal anomalies, gestational diabetes, group B strep, cystitis, pyelonephritis, cholecystitis, hypertension, preeclampsia, oligohydramnios, polyhydramnios, and premature rupture of membranes, etc. Leaking of clear fluid from the vagina with back pain and elevated BP is associated with premature rupture of membranes, a second trimester complication of pregnancy. Preconception is prior to becoming pregnant; postpartum is after delivery of the infant; and first trimester is not associated with premature rupture of the membranes usually.

A female patient comes to the clinic after missing one menstrual period. She lives in a house beneath electrical power lines which is located near an oil field. She drinks two caffeinated beverages a day, is a daily beer drinker, and has not stopped eating sweets. She takes a multivitamin and exercises daily. She denies drug use. Which finding in the history has the greatest implication for this patient's plan of care? a. Electrical power lines are a potential hazard to the woman and her fetus. b. Living near an oil field may mean the water supply is polluted. c. Alcohol exposure should be avoided during pregnancy due to teratogenicity. d. Eating sweets may cause gestational diabetes or miscarriage.

ANS:C Stages of development include ovum, embryonic, and fetal. The beginning of the fourth week to the end of the eighth week comprise the embryonic period. Teratogenicity is a major concern because all external and internal structures are developing in the embryonic period. A pregnant woman should avoid exposure to all potential toxins during pregnancy, especially alcohol, tobacco, radiation, and infections during embryonic development. Living in a house beneath power lines is not the greatest implication in this patient's plan of care as there are no definite risks to the developing fetus. Living near an oil field has no definite risks to the fetus. Eating sweets may contribute to maternal obesity, large for gestational age fetus, and maternal gestational diabetes but does not have the immediate implication of a daily beer drinker which can cause fetal alcohol syndrome.

At 1 minute after birth, the nurse assesses the newborn to assign an Apgar score. The apical heart rate is 110 bpm, and the infant is crying vigorously with the limbs flexed. The infant's trunk is pink, but the hands and feet are blue. What is the correct Apgar score for this infant? a. 7 c. 9 b. 8 d. 10

ANS:C The Apgar score is 9 because 1 point is deducted from the total score of 10 for the infant's blue hands and feet. The baby received 2 points for each of the categories except color. Because the infant's hands and feet were blue, this category is given a grade of 1.

According to standard professional thinking, nurses should auscultate the fetal heart rate (FHR): a. Every 15 minutes in the active phase of the first stage of labor in the absence of risk factors. b. Every 20 minutes in the second stage, regardless of whether risk factors are present. c. Before and after ambulation and rupture of membranes. d. More often in a woman's first pregnancy.

ANS:C The FHR should be auscultated before and after administration of medications and induction of anesthesia. In the active phase of the first stage of labor, the FHR should be auscultated every 30 minutes if no risk factors are involved; with risk factors it should be auscultated every 15 minutes. In the second stage of labor the FHR should be auscultated every 15 minutes if no risk factors are involved; with risk factors it should be auscultated every 5 minutes. The fetus of a first-time mother is automatically at greater risk.

Which description of the phases of the second stage of labor is accurate? a. Latent phase: Feeling sleepy, fetal station 2+ to 4+, duration 30 to 45 minutes b. Active phase: Overwhelmingly strong contractions, Ferguson reflux activated, duration 5 to 15 minutes c. Descent phase: Significant increase in contractions, Ferguson reflux activated, average duration varied d. Transitional phase: Woman "laboring down," fetal station 0, duration 15 minutes

ANS:C The descent phase begins with a significant increase in contractions; the Ferguson reflex is activated, and the duration varies, depending on a number of factors. The latent phase is the lull, or "laboring down," period at the beginning of the second stage. It lasts 10 to 30 minutes on average. The second stage of labor has no active phase. The transition phase is the final phase in the second stage of labor; contractions are strong and painful.

To provide safe care for the woman, the nurse understands that which condition is a contraindication for an amniotomy? a. Dilation less than 3 cm c. -2 station b. Cephalic presentation d. Right occiput posterior position

ANS:C The dilation of the cervix must be great enough to determine labor. The presenting part of the fetus should be engaged and well applied to the cervix before the procedure in order to prevent cord prolapse. Amniotomy is deferred if the presenting part is higher in the pelvis. ROP indicates a cephalic presentation, which is appropriate for an amniotomy.

When assessing the fetus using Leopold maneuvers, the nurse feels a round, firm, movable fetal part in the fundal portion of the uterus and a long, smooth surface in the mother's right side close to midline. What is the likely position of the fetus? a. ROA c. RSA b. LSP d. LOA

ANS:C The fetus is positioned anteriorly in the right side of the maternal pelvis with the sacrum as the presenting part. RSA is the correct three-letter abbreviation to indicate this fetal position. The first letter indicates the presenting part in either the right or left side of the maternal pelvis. The second letter indicates the anatomic presenting part of the fetus. The third letter stands for the location of the presenting part in relation to the anterior, posterior, or transverse portion of the maternal pelvis. Palpation of a round, firm fetal part in the fundal portion of the uterus would be the fetal head, indicating that the fetus is in a breech position with the sacrum as the presenting part in the maternal pelvis. Palpation of the fetal spine along the mother's right side denotes the location of the presenting part in the mother's pelvis. The ability to palpate the fetal spine indicates that the fetus is anteriorly positioned in the maternal pelvis.

The nurse providing care to a woman in labor should understand that cesarean birth: a. Is declining in frequency in the twenty-first century in the United States. b. Is more likely to be performed for poor women in public hospitals who do not receive the nurse counseling as do wealthier clients. c. Is performed primarily for the benefit of the fetus. d. Can be either elected or refused by women as their absolute legal right.

ANS:C The most common indications for cesarean birth are danger to the fetus related to labor and birth complications. Cesarean births are increasing in the United States in this century. Wealthier women who have health insurance and who give birth in a private hospital are more likely to experience cesarean birth. A woman's right to elect cesarean surgery is in dispute, as is her right to refuse it if in doing so she endangers the fetus. Legal issues are not absolutely clear.

A woman in labor has just received an epidural block. The most important nursing intervention is to: a. Limit parenteral fluids. b. Monitor the fetus for possible tachycardia. c. Monitor the maternal blood pressure for possible hypotension. d. Monitor the maternal pulse for possible bradycardia.

ANS:C The most important nursing intervention for a woman who has received an epidural block is to monitor the maternal blood pressure frequently for signs of hypotension. Intravenous fluids are increased for a woman receiving an epidural, to prevent hypotension. The nurse observes for signs of fetal bradycardia. The nurse monitors for signs of maternal tachycardia secondary to hypotension.

With regard to factors that affect how the fetus moves through the birth canal, nurses should be aware that: a. The fetal attitude describes the angle at which the fetus exits the uterus. b. Of the two primary fetal lies, the horizontal lie is that in which the long axis of the fetus is parallel to the long axis of the mother. c. The normal attitude of the fetus is called general flexion. d. The transverse lie is preferred for vaginal birth.

ANS:C The normal attitude of the fetus is general flexion. The fetal attitude is the relation of fetal body parts to one another. The horizontal lie is perpendicular to the mother; in the longitudinal (or vertical) lie the long axes of the fetus and the mother are parallel. Vaginal birth cannot occur if the fetus stays in a transverse lie.

The uterine contractions of a woman early in the active phase of labor are assessed by an internal uterine pressure catheter (IUPC). The nurse notes that the intrauterine pressure at the peak of the contraction ranges from 65 to 70 mm Hg and the resting tone range is 6 to 10 mm Hg. The uterine contractions occur every 3 to 4 minutes and last an average of 55 to 60 seconds. On the basis of this information, the nurse should: a. Notify the woman's primary health care provider immediately. b. Prepare to administer an oxytocic to stimulate uterine activity. c. Document the findings because they reflect the expected contraction pattern for the active phase of labor. d. Prepare the woman for the onset of the second stage of labor.

ANS:C The nurse is responsible for monitoring the uterine contractions to ascertain whether they are powerful and frequent enough to accomplish the work of expelling the fetus and the placenta. In addition, the nurse would document these findings in the client's medical record. This labor pattern indicates that the client is in the active phase of the first stage of labor. Nothing indicates a need to notify the primary care provider at this time. Oxytocin augmentation is not needed for this labor pattern; this contraction pattern indicates adequate active labor. Her contractions eventually will become stronger, last longer, and come closer together during the transition phase of the first stage of labor. The transition phase precedes the second stage of labor, or delivery of the fetus.

Which action is correct when palpation is used to assess the characteristics and pattern of uterine contractions? a. Place the hand on the abdomen below the umbilicus and palpate uterine tone with the fingertips. b. Determine the frequency by timing from the end of one contraction to the end of the next contraction. c. Evaluate the intensity by pressing the fingertips into the uterine fundus. d. Assess uterine contractions every 30 minutes throughout the first stage of labor.

ANS:C The nurse or primary care provider may assess uterine activity by palpating the fundal section of the uterus using the fingertips. Many women may experience labor pain in the lower segment of the uterus that may be unrelated to the firmness of the contraction detectable in the uterine fundus. The frequency of uterine contractions is determined by palpating from the beginning of one contraction to the beginning of the next contraction. Assessment of uterine activity is performed in intervals based on the stage of labor. As labor progresses this assessment is performed more frequently.

A woman has requested an epidural for her pain. She is 5 cm dilated and 100% effaced. The baby is in a vertex position and is engaged. The nurse increases the woman's intravenous fluid for a preprocedural bolus. She reviews her laboratory values and notes that the woman's hemoglobin is 12 g/dL, hematocrit is 38%, platelets are 67,000, and white blood cells (WBCs) are 12,000/mm3. Which factor would contraindicate an epidural for the woman? a. She is too far dilated. c. She has thrombocytopenia. b. She is anemic. d. She is septic.

ANS:C The platelet count indicates a coagulopathy, specifically, thrombocytopenia (low platelets), which is a contraindication to epidural analgesia/anesthesia. Typically epidural analgesia/anesthesia is used in the laboring woman when a regular labor pattern has been achieved, as evidenced by progressive cervical change. The laboratory values show that the woman's hemoglobin and hematocrit are in the normal range and show a slight increase in the WBC count that is not uncommon in laboring women.

With regard to the turns and other adjustments of the fetus during the birth process, known as the mechanism of labor, nurses should be aware that: a. The seven critical movements must progress in a more or less orderly sequence. b. Asynclitism sometimes is achieved by means of the Leopold maneuver. c. The effects of the forces determining descent are modified by the shape of the woman's pelvis and the size of the fetal head. d. At birth the baby is said to achieve "restitution" (i.e., a return to the C-shape of the womb).

ANS:C The size of the maternal pelvis and the ability of the fetal head to mold also affect the process. The seven identifiable movements of the mechanism of labor occur in combinations simultaneously, not in precise sequences.Asynclitism is the deflection of the baby's head; the Leopold maneuver is a means of judging descent by palpating the mother's abdomen. Restitution is the rotation of the baby's head after the infant is born.

What is an advantage of external electronic fetal monitoring? a. The ultrasound transducer can accurately measure short-term variability and beat-to-beat changes in the fetal heart rate. b. The tocotransducer can measure and record the frequency, regularity, intensity, and approximate duration of uterine contractions (UCs). c. The tocotransducer is especially valuable for measuring uterine activity during the first stage of labor. d. Once correctly applied by the nurse, the transducer need not be repositioned even when the woman changes positions.

ANS:C The tocotransducer is especially valuable for measuring uterine activity during the first stage of labor, particularly when the membranes are intact. Short-term changes cannot be measured with this technology. The tocotransducer cannot measure and record the intensity of UCs. The transducer must be repositioned when the woman or fetus changes position.

A woman in labor is breathing into a mouthpiece just before the start of her regular contractions. As she inhales, a valve opens, and gas is released. She continues to inhale the gas slowly and deeply until the contraction starts to subside. When the inhalation stops, the valve closes. This procedure is: a. Not used much anymore. b. Likely to be used in the second stage of labor but not in the first stage. c. An application of nitrous oxide. d. A prelude to cesarean birth.

ANS:C This is an application of nitrous oxide, which could be used in either the first or second stage of labor (or both) as part of the preparation for a vaginal birth. Nitrous oxide is self-administered and found to be very helpful.

The nurse teaches a pregnant woman about the characteristics of true labor contractions. The nurse evaluates the woman's understanding of the instructions when she states, "True labor contractions will: a. Subside when I walk around." b. Cause discomfort over the top of my uterus." c. Continue and get stronger even if I relax and take a shower." d. Remain irregular but become stronger."

ANS:C True labor contractions occur regularly, becoming stronger, lasting longer, and occurring closer together. They may become intense during walking and continue despite comfort measures. Typically true labor contractions are felt in the lower back, radiating to the lower portion of the abdomen. During false labor, contractions tend to be irregular and felt in the abdomen above the navel. Typically the contractions often stop with walking or a change of position.

The maternity nurse understands that as the uterus contracts during labor, maternal-fetal exchange of oxygen and waste products: a. Continues except when placental functions are reduced. b. Increases as blood pressure decreases. c. Diminishes as the spiral arteries are compressed. d. Is not significantly affected.

ANS:C Uterine contractions during labor tend to decrease circulation through the spiral electrodes and subsequent perfusion through the intervillous space. The maternal blood supply to the placenta gradually stops with contractions. The exchange of oxygen and waste products decreases. The exchange of oxygen and waste products is affected by contractions.

The nurse providing care for the laboring woman should understand that late fetal heart rate (FHR) decelerations are the result of: a. Altered cerebral blood flow. c. Uteroplacental insufficiency. b. Umbilical cord compression. d. Meconium fluid.

ANS:C Uteroplacental insufficiency would result in late decelerations in the FHR. Altered fetal cerebral blood flow would result in early decelerations in the FHR. Umbilical cord compression would result in variable decelerations in the FHR. Meconium-stained fluid may or may not produce changes in the fetal heart rate, depending on the gestational age of the fetus and whether other causative factors associated with fetal distress are present.

A woman is having her first child. She has been in labor for 15 hours. Two hours ago her vaginal examination revealed the cervix to be dilated to 5 cm and 100% effaced, and the presenting part was at station 0. Five minutes ago her vaginal examination indicated that there had been no change. What abnormal labor pattern is associated with this description? a. Prolonged latent phase c. Arrest of active phase b. Protracted active phase d. Protracted descent

ANS:C With an arrest of the active phase, the progress of labor has stopped. This client has not had any anticipated cervical change, thus indicating an arrest of labor. In the nulliparous woman a prolonged latent phase typically would last more than 20 hours. A protracted active phase, the first or second stage of labor, would be prolonged (slow dilation). With protracted descent, the fetus would fail to descend at an anticipated rate during the deceleration phase and second stage of labor.

A primigravida at 40 weeks of gestation is having uterine contractions every 1.5 to 2 minutes and says that they are very painful. Her cervix is dilated 2 cm and has not changed in 3 hours. The woman is crying and wants an epidural. What is the likely status of this woman's labor? a. She is exhibiting hypotonic uterine dysfunction. b. She is experiencing a normal latent stage. c. She is exhibiting hypertonic uterine dysfunction. d. She is experiencing pelvic dystocia.

ANS:C Women who experience hypertonic uterine dysfunction, or primary dysfunctional labor, often are anxious first-time mothers who are having painful and frequent contractions that are ineffective at causing cervical dilation or effacement to progress. With hypotonic uterine dysfunction, the woman initially makes normal progress into the active stage of labor; then the contractions become weak and inefficient or stop altogether. The contraction pattern seen in this woman signifies hypertonic uterine activity. Typically uterine activity in this phase occurs at 4- to 5-minute intervals lasting 30 to 45 seconds. Pelvic dystocia can occur whenever contractures of the pelvic diameters reduce the capacity of the bony pelvis, including the inlet, midpelvis, outlet, or any combination of these planes.

In her birth plan, a woman requests that she be allowed to use the new whirlpool bath during labor. When implementing this woman's request, the nurse should: A. Assist the woman to maintain a reclining position when in the tub B. Tell the woman she will need to leave the tub as soon as her membranes rupture C. Begin hydrotherapy when the woman is in active labor (approximately 5 cm) D. Limit her to no longer than 1 hour in the tub

ANS:C is correct; beginning in active labor reduces the risk that labor will slow down; there is no limit to the time a woman can stay in the tub; woman can and should change her position while in the bath, using lateral and hand-and-knees positions when indicated; there is no evidence indicating an increased risk for infection when membranes are ruptured; however, tubs must be carefully cleansed; contractions may slow if bath is begun in the latent phase.

The baseline fetal heart rate (FHR) is the average rate during a 10-minute segment. Changes in FHR are categorized as periodic or episodic. These patterns include both accelerations and decelerations. The labor nurse is evaluating the patient's most recent 10-minute segment on the monitor strip and notes a late deceleration. This is likely to be caused by which physiologic alteration (Select all that apply)? a. Spontaneous fetal movement b. Compression of the fetal head c. Placental abruption d. Cord around the baby's neck e. Maternal supine hypotension

ANS:C,E Late decelerations are almost always caused by uteroplacental insufficiency. Insufficiency is caused by uterine tachysystole, maternal hypotension, epidural or spinal anesthesia, IUGR, intraamniotic infection, or placental abruption. Spontaneous fetal movement, vaginal examination, fetal scalp stimulation, fetal reaction to external sounds, uterine contractions, fundal pressure and abdominal palpation are all likely to cause accelerations of the FHR. Early decelerations are most often the result of fetal head compression and may be caused by uterine contractions, fundal pressure, vaginal examination, and placement of an internal electrode. A variable deceleration is likely caused by umbilical cord compression. This may happen when the umbilical cord is around the baby's neck, arm, leg, or other body part or when there is a short cord, a knot in the cord, or a prolapsed cord.

A primigravida calls the hospital and tells a nurse on the labor unit that she knows she is in labor. The nurse's initial response is: A. "Tell me why you know that you are in labor." B. "How far do you live from the hospital?" C. "When is your expected date of birth?" D. "Have your membranes ruptured."

ANS:Choice A is correct; although choices B, C, and D are all important questions, the first question should gather information regarding whether or not the woman is in labor

A woman's labor is being suppressed using IV magnesium sulfate. Which measure should be implemented during the infusion? A. Limit intravenous fluid intake to 125 mL/hour. B. Discontinue infusion if maternal respirations are less than 14 breaths/minute. C. Ensure that indomethacin is available should toxicity occur. D. Assist woman to maintain a comfortable semirecumbent position.

ANS:Choice A is correct; magnesium sulfate is a CNS depressant; woman should alternate lateral positions to decrease pressure on cervix, which could stimulate uterine contractions; calcium gluconate would be used if toxicity occurs; infusion should be discontin-ued if respiratory rate is less than 12 breaths/minute

A nurse caring for a pregnant woman suspected of being in preterm labor recognizes which sign as diagnostic of preterm labor? A. Cervical dilation of at least 2 cm B. Uterine contractions occurring every 15 minutes C. Spontaneous rupture of the membranes D. Presence of fetal fibronectin in cervical secretions

ANS:Choice A is correct; the definitive sign of preterm labor is significant change in the cervix; while uterine contractions do occur, they must occur at a frequency of more than six contractions per hour and cause sig-nificant changes in the cervix; fetal fibronectin indi-cates that the risk for preterm labor exists but it does not mean that preterm labor is occurring.

A woman's labor is being induced. The nurse assesses the woman's status and that of her fetus and the labor process just before an infusion increment of 2 milliunits/minute. The nurse discontinues the infusion and notifies the wom-an's primary health care provider if during this assessment she notes: A. Frequency of uterine contractions: every 11⁄2 minutes B. Variability of fetal heart rate (FHR): present C. Deceleration patterns: early decelerations noted with several contractions D. Intensity of uterine contractions at their peaks: 80 to 85 mm H

ANS:Choice A is correct; uterine contractions should not occur more frequently than five contractions in 10 minutes to allow for an adequate rest period between contractions; choices B, C, and D are all expected findings within the normal range

When evaluating the external fetal monitor tracing of a woman whose labor is being induced, the nurse identifies signs of persistent late deceleration patterns and begins intrauterine resuscitation interventions. Which choice indicates that the following appropriate interventions were implemented in the recommended order of priority? 1. Increase rate of maintenance IV solution. 2. Palpate uterus for tachysystole. 3. Discontinue oxytocin (Pitocin) infusion. 4. Change maternal position to a lateral position, and then elevate her legs if woman is hypotensive. 5. Administer oxygen at 8 to 10 L/minute by nonrebreather face mask. A. 2, 1, 5, 4, 3 B. 4, 1, 2, 3, 5 C. 5, 3, 4, 1, 2 D. 4, 5, 1, 2, 3

ANS:Choice B is correct; see Box 18-5 for the rationale.

A nulliparous woman is in the active phase of labor and her cervix has progressed to 6 cm dilation. The nurse caring for this woman evaluates the external monitor tracing and notes the following: decrease in FHR shortly after onset of several uterine contractions returning to baseline rate by the end of the contraction; shape is uniform. Based on these findings, the nurse should: A. Change the woman's position to her left side B. Document the finding on the woman's chart C. Notify the physician D. Perform a vaginal examination to check for cord prolapse

ANS:Choice B is correct; the pattern described is an early deceleration pattern, which is considered to be benign, requiring no action other than documen-tation of the finding; changing a woman's position and notifying the physician would be appropriate if abnormal (nonreassuring) signs such as late or vari-able decelerations were occurring; prolapse of cord is associated with variable decelerations as a result of cord compression.

A vaginal examination is performed on a multiparous woman who is in labor. The results of the examination were documented as 4 cm, 75%, +1, ROA. An accurate interpretation of these data is: A. Woman is in the latent phase of the first stage of labor B. Station is 1 cm above the ischial spines C. Presentation is cephalic D. Lie is transverse

ANS:Choice C is correct; O, or occiput, indicates a vertex presentation with the neck fully flexed and the occiput in the right anterior pelvic segment (R, A) of the woman's pelvis; the station is 2 cm below the ischial spines (+2); the woman is in the active phase of labor, as indicated by 4 cm of dilation, and effacement is 75%; the lie is longitudinal because the head (cephalic/vertex) is presenting.

A woman is in active labor. On spontaneous rupture of her membranes, the nurse caring for this woman notices variable deceleration patterns during evaluation of the monitor tracing. When preparing to perform a vaginal exam-ination, the nurse observes a small section of the umbilical cord protruding from the vagina. What should the nurse do next? A. Increase the IV drip rate. B. Administer oxygen to the woman via mask at 8 to 10 L/minute. C. Place a sterile gloved hand into the vagina and hold the presenting part off the cord while calling for assistance. D. Wrap the cord loosely with a sterile towel saturated with warm normal saline.

ANS:Choice C is correct; although choices A, B, and D are appropriate actions along with changes in her position, removing pressure from the cord to preserve perfusion is the priority; see Emergency—Prolapsed Umbilical Cord box.

A physician has ordered that dinoprostone (Cervidil) be administered to ripen a pregnant woman's cervix in prepa-ration for an induced labor. In fulfilling this order, the nurse should: A. Insert the dinoprostone in the cervical canal just below the internal os B. Tell the woman to remain in bed for at least 15 minutes C. Observe the woman for signs of uterine tachysystole D. Remove the dinoprostone as soon as the woman begins to experience uterine contrac

ANS:Choice C is correct; it is inserted in the posterior vaginal fornix; the woman should remain in bed for 2 hours; caution should be used if the woman has asthma; therefore ensure that physician is aware of asthma; the insert is removed for severe side effects such as uterine tachysystole; dinoprostone (Cervidil) often stimulates contractions and may even induce the onset of labor, eliminating or reducing the need for oxytocin (Pitocin); it should be removed after 12 hours or with onset of active labor.

A woman's amniotic membranes have apparently ruptured. The nurse assesses the fluid to determine its characteris-tics and confirm membrane rupture. An expected assessment finding of membrane rupture is: A. pH 5.5 B. Absence of ferning C. Pale, clear fluid with white flecks D. Strong odor

ANS:Choice C is correct; pH of amniotic fluid is alkaline at 6.5 or higher, ferning is noted when examining fluid with a microscope, and the fluid is relatively odorless; a strong odor is strongly suggestive of infection; white flecks indicate vernix caseosa.

A laboring woman's temperature is elevated as a result of an upper respiratory infection. The FHR pattern that reflects maternal fever is: A. Diminished variability B. Variable decelerations C. Tachycardia D. Early decelerations

ANS:Choice C is correct; the FHR increases as the mater-nal core body temperature elevates; therefore tachy-cardia is the pattern exhibited; it is often a clue of intrauterine infection because maternal fever is often the first sign; diminished variability reflects hypoxia, variable decelerations are characteristic of cord com-pression, and early decelerations are characteristic of head compression by the cervi

A laboring woman's vaginal examination reveals the following: 3 cm, 50%, LSA, 0. The nurse caring for this woman should: A. Place the ultrasound transducer in the left lower quadrant of the woman's abdomen B. Recognize that passage of meconium would be a definitive sign of fetal distress C. Expect the progress of fetal descent to be slower than usual D. Assist the woman into a knee-chest position for each contraction

ANS:Choice C is correct; the presentation of this fetus is breech; the soft buttocks are a less efficient dilating wedge than the fetal head; therefore labor may be slower; the ultrasound transducer should be placed to the left of the umbilicus at a level at or above it; pas-sage of meconium is an expected finding as a result of pressure on the abdomen during descent; knee-chest position would be used for occipitoposterior positio

A nulliparous woman experiencing a postterm pregnancy is admitted for labor induction. Assessment reveals a Bishop score of 9. The nurse should: A. Call the woman's primary health care provider to order a cervical ripening agent B. Mix 20 units of oxytocin (Pitocin) in 500 mL of 5% glucose in water C. Piggyback the oxytocin solution into the port nearest the drip chamber of the primary IV tubing D. Begin the infusion at a rate of 1 milliunit/minute as determined by the induction protoco

ANS:Choice D is correct; a Bishop score of 9 indicates that the cervix is already sufficiently ripe for successful induction; it is currently recommended that 30 units of oxytocin (Pitocin) be mixed in 500 mL of an elec-trolyte solution such as Ringers lactate; the oxytocin solution is piggybacked at the proximal port (port nearest the insertion site).

A laboring woman's uterine contractions are being internally monitored. When evaluating the monitor tracing, which finding is a source of concern and requires further assessment? A. Frequency every 21⁄2 to 3 minutes B. Duration of 80 to 85 seconds C. Intensity during a uterine contraction of 85 to 90 mm Hg D. Average resting pressure of 20 to 25 mm Hg

ANS:Choice D is correct; the average resting tone during labor should be 10 mm Hg; choices A, B, and C are all findings within the expected ranges.

A nurse is caring for a pregnant woman at 30 weeks of gestation in preterm labor. The woman's physician orders betamethasone 12 mg IM for two doses, with the first dose to begin at 11 am. In implementing this order the nurse should: A. Consult the physician, because the dose is too high B. Explain to the woman that this medication will reduce her heart rate and help her to breathe easier C. Prepare to administer the medication intravenously between contractions D. Schedule the second dose for 11 am on the next day

ANS:Choice D is correct; the dosage is correct at 12 mg × 2 doses; this medication will stimulate her baby's lungs to produce surfactant and help the baby to breathe more easily should birth occur; dosages should be spaced 24 hours apart; therefore the second dose should be given at 11 am on the next day; this medication is administered intramuscularly.

When admitting a primigravida to the labor unit, the nurse observes for signs that indicate that the woman is in true labor and should be admitted. The nurse recognizes which of the following signs as indicative of true labor? (Circle all that apply.) A. Woman reports that her contractions seem stronger since she walked from the car to her room on the labor unit. B. Cervix feels soft and is 50% effaced. C. Woman perceives pain to be in her back or abdomen above the level of the navel. D. Fetus is engaged in the pelvis at zero station. E. Cervix is in the posterior position. F. Woman continues to feel her contractions intensify following a back rub with a use of effleurage.

ANS:Choices A, B, D, and E are correct; pain of true labor is usually felt in the lower back radiating to the lower portion of the abdomen.

An anesthesiologist is preparing to begin a continuous epidural block using a combination local anesthetic and opioid analgesic as a pain relief measure for a laboring woman. Nursing measures related to this type of nerve block include: (Circle all that apply.) A. Assist the woman into a modified Sims position or upright position with back curved for administration of the block B. Alternate her position from side to side every hour C. Assess the woman for headaches, because they commonly occur in the postpartum period if an epidural is used for labor D. Assist the woman to urinate at least every 2 hours during labor to prevent bladder distention E. Prepare the woman for use of forceps-or vacuum-assisted birth, because she will be unable to bear down F. Assess blood pressure frequently, because severe hypotension can occur

ANS:Choices A, B, D, and F are correct; position with a curved back separates the vertebrae and facilitates administration of the anesthetic; alternating lateral positions after administration will prevent supine hypotension; she should be encouraged to empty her bladder every 2 hours; because the dura is not punc-tured, there is no leakage of cerebrospinal fluid, making spinal headache rare; the dosage can be adjusted to al-low her to push during the second stage of labor.

External electronic fetal monitoring will be used for a woman just admitted to the labor unit in active labor. Guidelines that the nurse should follow when implementing this form of monitoring are to: (Circle all that apply.) A. Use Leopold maneuvers to determine correct placement of the tocotransducer B. Assist the woman to maintain a dorsal recumbent position to ensure accurate monitor tracings that can be evaluated easily C. Apply contact gel to the ultrasound transducer prior to application over the point of maximum intensity of the FHR D. Reposition the tocotransducer when the fetus changes its position E. Caution the woman to avoid effleurage when the transducers are in place F. Palpate the fundus periodically to estimate the intensity of the uterine contractions

ANS:Choices C and F are correct; Leopold maneuvers are used to locate the PMI for correct placement of the ultrasound transducer; the tocotransducer is always placed over the fundus; the ultrasound transducer's position needs to change with fetal movement; the tocotransducer cannot assess intensity of contrac-tions; therefore periodic palpation is an essential assessment measure; effleurage can be used around the transducers or on other parts of the woman's body.

Which basic type of pelvis includes the correct description and percentage of occurrence in women? a. Gynecoid: classic female; heart shaped; 75% b. Android: resembling the male; wider oval; 15% c. Anthropoid: resembling the ape; narrower; 10% d. Platypelloid: flattened, wide, shallow; 3%

ANS:D A platypelloid pelvis is flattened, wide, and shallow; about 3% of women have this shape. The gynecoid shape is the classical female shape, slightly ovoid and rounded; about 50% of women have this shape. An android, or malelike, pelvis is heart shaped; about 23% of women have this shape. An anthropoid, or apelike, pelvis is oval and wider; about 24% of women have this shape.

The most common cause of decreased variability in the fetal heart rate (FHR) that lasts 30 minutes or less is: a. Altered cerebral blood flow. c. Umbilical cord compression. b. Fetal hypoxemia. d. Fetal sleep cycles.

ANS:D A temporary decrease in variability can occur when the fetus is in a sleep state. These sleep states do not usually last longer than 30 minutes. Altered fetal cerebral blood flow would result in early decelerations in the FHR. Fetal hypoxemia would be evidenced by tachycardia initially and then bradycardia. A persistent decrease or loss of FHR variability may be seen. Umbilical cord compression would result in variable decelerations in the FHR.

The obstetric nurse is preparing the patient for an emergency cesarean birth, with no time to administer spinal anesthesia. The nurse is aware and prepared for the greatest risk of administering general anesthesia to the patient. This risk is: a. Respiratory depression. c. Inadequate muscle relaxation. b. Uterine relaxation. d. Aspiration of stomach contents.

ANS:D Aspiration of acidic gastric contents with possible airway obstruction is a potentially fatal complication of general anesthesia. Respirations can be altered during general anesthesia, and the anesthesiologist will take precautions to maintain proper oxygenation. Uterine relaxation can occur with some anesthesia; however, this can be monitored and prevented. Inadequate muscle relaxation can be improved with medication.

When a nulliparous woman telephones the hospital to report that she is in labor, the nurse initially should: a. Tell the woman to stay home until her membranes rupture. b. Emphasize that food and fluid intake should stop. c. Arrange for the woman to come to the hospital for labor evaluation. d. Ask the woman to describe why she believes she is in labor.

ANS:D Assessment begins at the first contact with the woman, whether by telephone or in person. By asking the woman to describe her signs and symptoms, the nurse can begin the assessment and gather data. The amniotic membranes may or may not spontaneously rupture during labor. The client may be instructed to stay home until the uterine contractions become strong and regular. The nurse may want to discuss the appropriate oral intake for early labor such as light foods or clear liquids, depending on the preference of the client or her primary health care provider. Before instructing the woman to come to the hospital, the nurse should initiate the assessment during the telephone interview.

A woman at 26 weeks of gestation is being assessed to determine whether she is experiencing preterm labor. What finding indicates that preterm labor is occurring? a. Estriol is not found in maternal saliva. b. Irregular, mild uterine contractions are occurring every 12 to 15 minutes. c. Fetal fibronectin is present in vaginal secretions. d. The cervix is effacing and dilated to 2 cm.

ANS:D Cervical changes such as shortened endocervical length, effacement, and dilation are predictors of imminent preterm labor. Changes in the cervix accompanied by regular contractions indicate labor at any gestation. Estriol is a form of estrogen produced by the fetus that is present in plasma at 9 weeks of gestation. Levels of salivary estriol have been shown to increase before preterm birth. Irregular, mild contractions that do not cause cervical change are not considered a threat. The presence of fetal fibronectin in vaginal secretions between 24 and 36 weeks of gestation could predict preterm labor, but it has only a 20% to 40% positive predictive value. Of more importance are other physiologic clues of preterm labor such as cervical changes.

. An 18-year-old pregnant woman, gravida 1, is admitted to the labor and birth unit with moderate contractions every 5 minutes that last 40 seconds. The woman states, "My contractions are so strong that I don't know what to do with myself." The nurse should: a. Assess for fetal well-being. b. Encourage the woman to lie on her side. c. Disturb the woman as little as possible. d. Recognize that pain is personalized for each individual.

ANS:D Each woman's pain during childbirth is unique and is influenced by a variety of physiologic, psychosocial, and environmental factors. A critical issue for the nurse is how support can make a difference in the pain of the woman during labor and birth. Assessing for fetal well-being includes no information that would indicate fetal distress or a logical reason to be overly concerned about the well-being of the fetus. The left lateral position is used to alleviate fetal distress, not maternal stress. The nurse has an obligation to provide physical, emotional, and psychosocial care and support to the laboring woman. This client clearly needs support.

In order to care for obstetric patients adequately, the nurse understands that labor contractions facilitate cervical dilation by: a. Contracting the lower uterine segment. b. Enlarging the internal size of the uterus. c. Promoting blood flow to the cervix. d. Pulling the cervix over the fetus and amniotic sac.

ANS:D Effective uterine contractions pull the cervix upward at the same time that the fetus and amniotic sac are pushed downward. The contractions are stronger at the fundus. The internal size becomes smaller with the contractions; this helps to push the fetus down. Blood flow decreases to the uterus during a contraction.

In the current practice of childbirth preparation, emphasis is placed on: a. The Dick-Read (natural) childbirth method. b. The Lamaze (psychoprophylactic) method. c. The Bradley (husband-coached) method. d. Having expectant parents attend childbirth preparation in any or no specific method.

ANS:D Encouraging expectant parents to attend childbirth preparation class is most important because preparation increases a woman's confidence and thus her ability to cope with labor and birth. Although still popular, the "method" format of classes is being replaced with other offerings such as Hypnobirthing and Birthing from Within.

When assessing the relative advantages and disadvantages of internal and external electronic fetal monitoring, nurses comprehend that both: a. Can be used when membranes are intact. b. Measure the frequency, duration, and intensity of uterine contractions. c. May need to rely on the woman to indicate when uterine activity (UA) is occurring. d. Can be used during the antepartum and intrapartum periods.

ANS:D External monitoring can be used in both periods; internal monitoring can be used only in the intrapartum period. For internal monitoring the membranes must have ruptured, and the cervix must be sufficiently dilated. Internal monitoring measures the intensity of contractions; external monitoring cannot do this. With external monitoring, the woman may need to alert the nurse that UA is occurring; internal monitoring does not require this.

The RN at the Preconception Counseling Clinic takes a male history for infertility evaluation. Which finding has the greatest implication for this patient's care? a. Practice of nightly masturbation b. Primary anovulation c. High testosterone levels d. Impotence due to alcohol ingestion

ANS:D Factors affecting male infertility include impotence due to alcohol. Nightly masturbation and high testosterone levels do not have the greatest implication on male infertility in a patient with admitted alcohol issues. Primary anovulation refers to female infertility, so it is not a consideration for this question about male infertility.

Increasing the infusion rate of nonadditive intravenous fluids can increase fetal oxygenation primarily by: a. Maintaining normal maternal temperature. b. Preventing normal maternal hypoglycemia. c. Increasing the oxygen-carrying capacity of the maternal blood. d. Expanding maternal blood volume.

ANS:D Filling the mother's vascular system makes more blood available to perfuse the placenta and may correct hypotension. Increasing fluid volume may alter the maternal temperature only if she is dehydrated. Most intravenous fluids for laboring women are isotonic and do not provide extra glucose. Oxygen-carrying capacity is increased by adding more red blood cells.

With regard to the care management of preterm labor, nurses should be aware that: a. Because all women must be considered at risk for preterm labor and prediction is so hit-and-miss, teaching pregnant women the symptoms probably causes more harm through false alarms. b. Braxton Hicks contractions often signal the onset of preterm labor. c. Because preterm labor is likely to be the start of an extended labor, a woman with symptoms can wait several hours before contacting the primary caregiver. d. The diagnosis of preterm labor is based on gestational age, uterine activity, and progressive cervical change.

ANS:D Gestational age of 20 to 37 weeks, uterine contractions, and a cervix that is 80% effaced or dilated 2 cm indicates preterm labor. It is essential that nurses teach women how to detect the early symptoms of preterm labor. Braxton Hicks contractions resemble preterm labor contractions, but they are not true labor. Waiting too long to see a health care provider could result in not administering essential medications. Preterm labor is not necessarily long-term labor.

Which statement correctly describes the effects of various pain factors? a. Higher prostaglandin levels arising from dysmenorrhea can blunt the pain of childbirth. b. Upright positions in labor increase the pain factor because they cause greater fatigue. c. Women who move around trying different positions are experiencing more pain. d. Levels of pain-mitigating b-endorphins are higher during a spontaneous, natural childbirth.

ANS:D Higher endorphin levels help women tolerate pain and reduce anxiety and irritability. Higher prostaglandin levels correspond to more severe labor pains. Upright positions in labor usually result in improved comfort and less pain. Moving freely to find more comfortable positions is important for reducing pain and muscle tension.

Nurses should be aware that the induction of labor: a. Can be achieved by external and internal version techniques. b. Is also known as a trial of labor (TOL). c. Is almost always done for medical reasons. d. Is rated for viability by a Bishop score.

ANS:D Induction of labor is likely to be more successful with a Bishop score of 9 or higher for first-time mothers and 5 or higher for veterans. Version is turning of the fetus to a better position by a physician for an easier or safer birth. A trial of labor is the observance of a woman and her fetus for several hours of active labor to assess the safety of vaginal birth. Two thirds of cases of induced labor are elective and are not done for medical reasons.

The standard of care for obstetrics dictates that an internal version may be used to manipulate the: a. Fetus from a breech to a cephalic presentation before labor begins. b. Fetus from a transverse lie to a longitudinal lie before cesarean birth. c. Second twin from an oblique lie to a transverse lie before labor begins. d. Second twin from a transverse lie to a breech presentation during vaginal birth.

ANS:D Internal version is used only during vaginal birth to manipulate the second twin into a presentation that allows it to be born vaginally. For internal version to occur, the cervix needs to be completely dilated.

For the labor nurse, care of the expectant mother begins with any or all of these situations, with the exception of: a. The onset of progressive, regular contractions. b. The bloody, or pink, show. c. The spontaneous rupture of membranes. d. Formulation of the woman's plan of care for labor.

ANS:D Labor care begins when progressive, regular contractions begin; the blood-tinged mucoid vaginal discharge appears; or fluid is discharged from the vagina. The woman and nurse can formulate their plan of care before labor or during treatment.

Which fetal heart rate (FHR) finding would concern the nurse during labor? a. Accelerations with fetal movement c. An average FHR of 126 beats/min b. Early decelerations d. Late decelerations

ANS:D Late decelerations are caused by uteroplacental insufficiency and are associated with fetal hypoxemia. They are considered ominous if persistent and uncorrected. Accelerations in the FHR are an indication of fetal well-being. Early decelerations in the FHR are associated with head compression as the fetus descends into the maternal pelvic outlet; they generally are not a concern during normal labor.

When using intermittent auscultation (IA) for fetal heart rate, nurses should be aware that: a. They can be expected to cover only two or three clients when IA is the primary method of fetal assessment. b. The best course is to use the descriptive terms associated with electronic fetal monitoring (EFM) when documenting results. c. If the heartbeat cannot be found immediately, a shift must be made to EFM. d. Ultrasound can be used to find the fetal heartbeat and reassure the mother if initial difficulty was a factor.

ANS:D Locating fetal heartbeats often takes time. Mothers can be reassured verbally and by the ultrasound pictures if ultrasound is used to help locate the heartbeat. When used as the primary method of fetal assessment, auscultation requires a nurse-to-client ratio of one to one. Documentation should use only terms that can be numerically defined; the usual visual descriptions of EFM are inappropriate.

Which method of pain management is safest for a gravida 3 para 2 admitted at 8 cm cervical dilation? a. Epidural anesthesia c. Spinal block b. Narcotics d. Breathing and relaxation techniques

ANS:D Nonpharmacologic methods of pain management may be the best option for a woman in advanced labor. It is unlikely that enough time remains to administer epidural or spinal anesthesia. A narcotic given at this time may reach its peak about the time of birth and result in respiratory depression in the newborn.

As a perinatal nurse you realize that a fetal heart rate that is tachycardic, is bradycardic, or has late decelerations or loss of variability is nonreassuring and is associated with: a. Hypotension. c. Maternal drug use. b. Cord compression. d. Hypoxemia.

ANS:D Nonreassuring heart rate patterns are associated with fetal hypoxemia. Fetal bradycardia may be associated with maternal hypotension. Fetal variable decelerations are associated with cord compression. Maternal drug use is associated with fetal tachycardia.

Which assessment is least likely to be associated with a breech presentation? a. Meconium-stained amniotic fluid b. Fetal heart tones heard at or above the maternal umbilicus c. Preterm labor and birth d. Post-term gestation

ANS:D Post-term gestation is not likely to be seen with a breech presentation. The presence of meconium in a breech presentation may result from pressure on the fetal wall as it traverses the birth canal. Fetal heart tones heard at the level of the umbilical level of the mother are a typical finding in a breech presentation because the fetal back would be located in the upper abdominal area. Breech presentations often occur in preterm births.

Which collection of risk factors most likely would result in damaging lacerations (including episiotomies)? a. A dark-skinned woman who has had more than one pregnancy, who is going through prolonged second-stage labor, and who is attended by a midwife b. A reddish-haired mother of two who is going through a breech birth c. A dark-skinned, first-time mother who is going through a long labor d. A first-time mother with reddish hair whose rapid labor was overseen by an obstetrician

ANS:D Reddish-haired women have tissue that is less distensible than that of darker-skinned women and therefore may have less efficient healing. First time mothers are also more at risk, especially with breech births, long second-stage labors, or rapid labors in which there is insufficient time for the perineum to stretch. The rate of episiotomies is higher when obstetricians rather than midwives attend births.

For a woman at 42 weeks of gestation, which finding would require further assessment by the nurse? a. Fetal heart rate of 116 beats/min b. Cervix dilated 2 cm and 50% effaced c. Score of 8 on the biophysical profile d. One fetal movement noted in 1 hour of assessment by the mother

ANS:D Self-care in a post-term pregnancy should include performing daily fetal kick counts three times per day. The mother should feel four fetal movements per hour. If fewer than four movements have been felt by the mother, she should count for 1 more hour. Fewer than four movements in that hour warrants evaluation. Normal findings in a 42-week gestation include fetal heart rate of 116 beats/min, cervix dilated 20 cm and 50% effaced, and a score of 8 on the biophysical profile.

While evaluating an external monitor tracing of a woman in active labor whose labor is being induced, the nurse notes that the fetal heart rate (FHR) begins to decelerate at the onset of several contractions and returns to baseline before each contraction ends. The nurse should: a. Change the woman's position. b. Discontinue the oxytocin infusion. c. Insert an internal monitor. d. Document the finding in the client's record.

ANS:D The FHR indicates early decelerations, which are not an ominous sign and do not require any intervention. The nurse should simply document these findings.

In order to evaluate the condition of the patient accurately during labor, the nurse should be aware that: a. The woman's blood pressure will increase during contractions and fall back to prelabor normal between contractions. b. Use of the Valsalva maneuver is encouraged during the second stage of labor to relieve fetal hypoxia. c. Having the woman point her toes will reduce leg cramps. d. The endogenous endorphins released during labor will raise the woman's pain threshold and produce sedation.

ANS:D The endogenous endorphins released during labor will raise the woman's pain threshold and produce sedation. In addition, physiologic anesthesia of the perineal tissues, caused by the pressure of the presenting part, decreases the mother's perception of pain. Blood pressure increases during contractions but remains somewhat elevated between them. Use of the Valsalva maneuver is discouraged during second stage labor because of a number of unhealthy outcomes, including fetal hypoxia. Pointing the toes can cause leg cramps, as can the process of labor itself.

The priority nursing intervention after an amniotomy should be to: a. Assess the color of the amniotic fluid. b. Change the patient's gown. c. Estimate the amount of amniotic fluid. d. Assess the fetal heart rate.

ANS:D The fetal heart rate must be assessed immediately after the rupture of the membranes to determine whether cord prolapse or compression has occurred. Secondary to FHR assessment, amniotic fluid amount, color, odor, and consistency is assessed. Dry clothing is important for patient comfort; however, it is not the top priority.

The factors that affect the process of labor and birth, known commonly as the five Ps, include all except: a. Passenger. c. Powers. b. Passageway. d. Pressure.

ANS:D The five Ps are passenger (fetus and placenta), passageway (birth canal), powers (contractions), position of the mother, and psychologic response.

If a woman complains of back labor pain, the nurse could best suggest that she: a. Lie on her back for a while with her knees bent. b. Do less walking around. c. Take some deep, cleansing breaths. d. Lean over a birth ball with her knees on the floor.

ANS:D The hands-and-knees position, with or without the aid of a birth ball, should help with the back pain. The supine position should be discouraged. Walking generally is encouraged.

A new mother asks the nurse when the "soft spot" on her son's head will go away. The nurse's answer is based on the knowledge that the anterior fontanel closes after birth by _____ months. a.2 b.8 c.12 d.18

ANS:D The larger of the two fontanels, the anterior fontanel, closes by 18 months after birth.

The nurse practicing in a labor setting knows that the woman most at risk for uterine rupture is: a. A gravida 3 who has had two low-segment transverse cesarean births. b. A gravida 2 who had a low-segment vertical incision for delivery of a 10-pound infant. c. A gravida 5 who had two vaginal births and two cesarean births. d. A gravida 4 who has had all cesarean births.

ANS:D The risk of uterine rupture increases for the patient who has had multiple prior births with no vaginal births. As the number of prior uterine incisions increases, so does the risk for uterine rupture. Low-segment transverse cesarean scars do not predispose the patient to uterine rupture.

A nurse may be called on to stimulate the fetal scalp: a. As part of fetal scalp blood sampling. b. In response to tocolysis. c. In preparation for fetal oxygen saturation monitoring. d. To elicit an acceleration in the fetal heart rate (FHR).

ANS:D The scalp can be stimulated using digital pressure during a vaginal examination. Fetal scalp blood sampling involves swabbing the scalp with disinfectant before a sample is collected. The nurse would stimulate the fetal scalp to elicit an acceleration of the FHR. Tocolysis is relaxation of the uterus. Fetal oxygen saturation monitoring involves the insertion of a sensor.

In evaluating the effectiveness of magnesium sulfate for the treatment of preterm labor, what finding would alert the nurse to possible side effects? a. Urine output of 160 mL in 4 hours b. Deep tendon reflexes 2+ and no clonus c. Respiratory rate of 16 breaths/min d. Serum magnesium level of 10 mg/dL

ANS:D The therapeutic range for magnesium sulfate management is 5 to 8 mg/dL. A serum magnesium level of 10 mg/dL could lead to signs and symptoms of magnesium toxicity, including oliguria and respiratory distress. Urine output of 160 mL in 4 hours, deep tendon reflexes 2+ with no clonus, and respiratory rate of 16 breaths/min are normal findings.

A multiparous woman has been in labor for 8 hours. Her membranes have just ruptured. The nurse's initial response would be to: a. Prepare the woman for imminent birth. b. Notify the woman's primary health care provider. c. Document the characteristics of the fluid. d. Assess the fetal heart rate and pattern.

ANS:D The umbilical cord may prolapse when the membranes rupture. The fetal heart rate and pattern should be monitored closely for several minutes immediately after ROM to ascertain fetal well-being, and the findings should be documented. Rupture of membranes (ROM) may increase the intensity and frequency of the uterine contractions, but it does not indicate that birth is imminent. The nurse may notify the primary care provider after ROM occurs and fetal well-being and the response to ROM have been assessed. The nurse's priority is to assess fetal well-being. The nurse should document the characteristics of the amniotic fluid, but the initial response is to assess fetal well-being and the response to ROM.

A primigravida at 39 weeks of gestation is observed for 2 hours in the intrapartum unit. The fetal heart rate has been normal. Contractions are 5 to 9 minutes apart, 20 to 30 seconds in duration, and of mild intensity. Cervical dilation is 1 to 2 cm and uneffaced (unchanged from admission). Membranes are intact. The nurse should expect the woman to be: a. Admitted and prepared for a cesarean birth. b. Admitted for extended observation. c. Discharged home with a sedative. d. Discharged home to await the onset of true labor.

ANS:D This situation describes a woman with normal assessments who is probably in false labor and will likely not deliver rapidly once true labor begins. There is no indication that further assessments or observations are indicated; therefore, the patient will be discharged along with instructions to return when contractions increase in intensity and frequency. Neither a cesarean birth nor a sedative is required at this time.

A primigravida at 39 weeks of gestation is observed for 2 hours in the intrapartum unit. The fetal heart rate has been normal. Contractions are 5 to 9 minutes apart, 20 to 30 seconds in duration, and of mild intensity. Cervical dilation is 1 to 2 cm and uneffaced (unchanged from admission). Membranes are intact. The nurse should expect the woman to be: a. Admitted and prepared for a cesarean birth. b. Admitted for extended observation. c. Discharged home with a sedative. d. Discharged home to await the onset of true labor.

ANS:D This situation describes a woman with normal assessments who is probably in false labor and will probably not deliver rapidly once true labor begins. These are all indications of false labor without fetal distress. There is no indication that further assessment or cesarean birth is indicated. The patient will likely be discharged; however, there is no indication that a sedative is needed.

A woman in the active phase of the first stage of labor is using a shallow pattern of breathing, which is about twice the normal adult breathing rate. She starts to complain about feeling lightheaded and dizzy and states that her fingers are tingling. The nurse should: a. Notify the woman's physician. b. Tell the woman to slow the pace of her breathing. c. Administer oxygen via a mask or nasal cannula. d. Help her breathe into a paper bag

ANS:D This woman is experiencing the side effects of hyperventilation, which include the symptoms of lightheadedness, dizziness, tingling of the fingers, or circumoral numbness. Having the woman breathe into a paper bag held tightly around her mouth and nose may eliminate respiratory alkalosis. This enables her to rebreathe carbon dioxide and replace the bicarbonate ion.

The physician has ordered nalbuphine hydrochloride (Nubain) 10 mg IV every 3 to 4 hours as needed for pain asso-ciated with labor. In fulfilling this order, the nurse knows that: A. This medication is a potent opioid agonist analgesic B. The dosage of the analgesic is too high for IV administration, necessitating a new order C. This is the analgesic of choice if the laboring woman is opioid dependent D. This analgesic is unlikely to cause significant maternal or fetal/neonatal respiratory depression

ANS:D is correct; nalbuphine hydrochloride (Nubain) is an opioid agonist-antagonist analgesic; a 10-mg dose is appropriate for IV administration; the antagonist component of this medication could precipitate with-drawal symptoms if the woman is opioid dependent.

When assessing a pregnant woman, the nurse is alert for factors associated with preterm labor. Which factor, if exhibited by this woman, increases her risk for spontaneous preterm labor and birth? (Circle all that apply.) A. Caucasian race B. Obstetric history of 3-0-2-0-1 C. History of bleeding at 20 weeks D. Currently being treated for second bladder infection in 2 months E. Multifetal gestation F. Body mass index (BMI) of 22 and height of 158 cm

Choices B, C, D, and E are correct; women who are underweight or overweight/obese; have multi-fetal pregnancies; are members of the non-Hispanic African-American race; or have a history of preterm birth, multiple miscarriages, infections of the geni-tourinary tract (including UTIs) and reproductive tract (such as bacterial vaginosis), and bleeding in the second trimester are at increased risk for preterm labor and birth.

A vaginal examination during labor reveals the following information: LOA, −1, 75%, 3 cm. An accurate interpretation of these data include: (Circle all that apply.) A. Attitude: flexed B. Station: 3 cm below the ischial spines C. Presentation: cephalic D. Lie: longitudinal E. Effacement: 75% complete F. Dilation: 9 cm more to reach full dilation

ANS: A, C, D, and E are correct; the presenting part is 1 cm above the ischial spines as indicated by the "-1"; attitude is flexion of head and neck, as indicated by the occiput ("O") as the presenting part; the lie is longitudinal, as indicated by the cephalic presentation; effacement is 75% complete and is 3 cm dilated, requiring 7 cm more for full dilation to 10 cm.

Changes occur as a woman progresses through labor. What maternal adaptations are expected during labor? (Circle all that apply.) A. Increase in both systolic and diastolic blood pressure during uterine contractions B. Decrease in white blood cell count C. Slight increase in baseline pulse and respiratory rates D. Decrease in gastric motility leading to nausea and vomiting, especially during the transition phase. E. Proteinuria 2+ F. Hyperglycemi

ANS: A, C, and D are correct; white blood cell count increases; a decrease in blood glucose and pro-teinuria of 1+ can be expected; see Box 16-2.

While developing an intrapartum care plan for the patient in early labor, it is important that the nurse recognize that psychosocial factors may influence a woman's experience of pain. These include (Select all that apply): a. Culture. b. Anxiety and fear. c. Previous experiences with pain. d. Intervention of caregivers. e. Support systems.

ANS: A,B,C,E Culture: a woman's sociocultural roots influence how she perceives, interprets, and responds to pain during childbirth. Some cultures encourage loud and vigorous expressions of pain, whereas others value self-control. The nurse should avoid praising some behaviors (stoicism) while belittling others (noisy expression). Anxiety and fear: extreme anxiety and fear magnify sensitivity to pain and impair a woman's ability to tolerate it. Anxiety and fear increase muscle tension in the pelvic area, which counters the expulsive forces of uterine contractions and pushing efforts. Previous experiences with pain: fear and withdrawal are a natural response to pain during labor. Learning about these normal sensations ahead of time helps a woman suppress her natural reactions of fear regarding the impending birth. If a woman previously had a long and difficult labor, she is likely to be anxious. She may also have learned ways to cope and may use these skills to adapt to the present labor experience. Support systems: an anxious partner is less able to provide help and support to a woman during labor. A woman's family and friends can be an important source of support if they convey realistic and positive information about labor and delivery. Although the intervention of caregivers may be necessary for the well-being of the woman and her fetus, some interventions add discomfort to the natural pain of labor (i.e., fetal monitor straps, intravenous lines).

A variety of medications may be administered to women during labor. Which medication is used to reduce anxiety, potentiate the effects of analgesics, and relieve nausea? A. Naloxone (Narcan) B. Metoclopramide (Reglan) C. Promethazine (Phenergan) D. Fentanyl citrate (Sublimaze)

ANS: B is correct; naloxone hydrochloride (Narcan) is an opioid antagonist; promethazine (Phenergan) is a phenothiazine that can decrease anxiety and nausea but it has been found to diminish the effectiveness of opioids in terms of pain relief; fentanyl citrate (Sublimaze) is an opioid agonist analgesic.

Following administration of fentanyl (Sublimaze) IV for pain associated with uterine contractions, a woman's labor progresses more rapidly than expected. The physician orders that a stat dose of naloxone (Narcan) 1 mg IV be ad-ministered to the woman to reverse respiratory depression in the newborn after its birth. In fulfilling this order, the nurse knows to: A. Question the route, because this medication should be administered orally B. Recognize that the dose is too low C. Assess the woman's level of pain, because it will return abruptly D. Observe the maternal pulse for bradycardia

ANS: C is correct; as an opioid antagonist it will reverse the effects of the opioid agonist analgesic administered for pain; maternal adverse reactions include hypotension or hypertension, tachycardia, hyperventilation, nausea, and vomiting; Narcan is administered intravenously at a dose of 0.4 to 2.0 mg depending on the amount of opioid agonist analge-sic given and the degree of CNS depression; it may be repeated at 2-to 3-minute intervals if needed for adequate reversal; Narcan is only given parenterally.

A nurse caring for women in labor should be aware of signs characterizing normal (reassuring) and abnormal (non-reassuring) FHR patterns. What would be characteristic of abnormal patterns? (Circle all that apply.) A. Moderate baseline variability B. Average baseline FHR of 100 beats/minute C. Acceleration of the FHR with movement D. Late deceleration patterns approximately every three or four contractions E. FHR of 170 beats/minute between contractions F. Early deceleration patterns when the cervix is dilated to 7 cm

ANS: Choices B, D, and E are correct; the baseline rate should be 110 to 160 beats/minute; accelerations should occur with fetal movement; no late decelera-tion pattern of any magnitude is normal (reassuring), especially if it is repetitive or uncorrectable; early deceleration patterns are expected findings when fetal head compression by the cervix occurs.

When instructing a group of primigravida women about the onset of labor, the nurse tells them to be alert for: A. Urinary retention B. Weight gain of 2 kg C. Quickening D. Energy surge

ANS:D is correct; quickening refers to the woman's first perception of fetal movement at 16 to 20 weeks of gestation; lightening accompanied by urinary fre-quency and weight loss of 0.5 to 1 kg occur to signal that the onset of labor is near; backache, stronger Braxton Hicks, and bloody show are also noted; see Box 16-1.

Before the physician performs an external version, the nurse should expect an order for a: a. Tocolytic drug. c. Local anesthetic. b. Contraction stress test (CST). d. Foley catheter.

ANS:A A tocolytic drug will relax the uterus before and during version, thus making manipulation easier. CST is used to determine the fetal response to stress. A local anesthetic is not used with external version. The bladder should be emptied; however, catheterization is not necessary.

When planning care for a laboring woman whose membranes have ruptured, the nurse recognizes that the woman's risk for _________________________ has increased. a. Intrauterine infection c. Precipitous labor b. Hemorrhage d. Supine hypotension

ANS:A When the membranes rupture, microorganisms from the vagina can ascend into the amniotic sac and cause chorioamnionitis and placentitis. Rupture of membranes (ROM) is not associated with fetal or maternal bleeding. Although ROM may increase the intensity of contractions and facilitate active labor, it does not result in precipitous labor. ROM has no correlation with supine hypotension.

What correctly matches the type of deceleration with its likely cause? a. Early deceleration—umbilical cord compression b. Late deceleration—uteroplacental inefficiency c. Variable deceleration—head compression d. Prolonged deceleration—cause unknown

ANS:B Late deceleration is caused by uteroplacental inefficiency. Early deceleration is caused by head compression. Variable deceleration is caused by umbilical cord compression. Prolonged deceleration has a variety of either benign or critical causes.

The least common cause of long, difficult, or abnormal labor (dystocia) is: a. Midplane contracture of the pelvis. b. Compromised bearing-down efforts as a result of pain medication. c. Disproportion of the pelvis. d. Low-lying placenta.

ANS:C The least common cause of dystocia is disproportion of the pelvis.

A woman at 27 weeks of gestation experiences some mild uterine cramping. Which actions should she take? (Circle all that apply.) A. Empty her bladder B. Call her nurse-midwife immediately C. Relax in a chair D. Drink two to three glasses of water or juice E. Palpate her uterus for 1 hour F. Resume the activity she was doing if the cramping subsides

ANS:Choices A, D, and E are correct; see Teaching for Self-Management—What to Do If Symptoms of Preterm Labor Occur box

An obstetric multipara with triplets is placed on bed rest at 24 weeks' gestation. Her perinatologist is managing intrauterine growth restriction with serial ultrasounds. This is an example of a. antenatal diagnostics. b. primary prevention. c. secondary prevention. d. tertiary prevention.

ANS:D, An example of tertiary prevention relating to reproductive health would be managing fetal intrauterine growth restriction by serial ultrasounds. This type of diagnostic maternal/fetal monitoring is performed to determine the best time for delivery due to potential fetal nutritional, circulatory, or pulmonary compromise. A cesarean section (operative delivery) may be performed if maternal or fetal conditions indicate that delivery is necessary. Antenatal diagnostics refers to prior to pregnancy. An example of primary prevention is teaching a high school class about reproductive health. An example of secondary prevention is prenatal care in the second trimester of pregnancy to prevent problems for the developing fetus.

A female college student is planning to become sexually active. She is considering birth control options and desires a method in which ovulation will be prevented. To prevent ovulation while reaching 99% effectiveness in preventing pregnancy, which option should be given the strongest consideration? a. Intrauterine device b. Coitus interruptus c. Natural family planning d. Oral contraceptive pills

ANS:D, Oral contraceptive pills prevent ovulation and are 99% effective in preventing pregnancy when taken as directed. Intrauterine devices, coitus interruptus, and natural family planning will not prevent ovulation while reaching 99% effectiveness in preventing pregnancy,so they are not recommended for this college student.

After induction of a spinal block in preparation for an elective cesarean birth, a woman's blood pressure decreases from 124/76 to 96/60. The nurse's initial action is to: A. Administer a vasopressor intravenously to raise the blood pressure B. Change the woman's position from supine to lateral C. Begin to administer oxygen by mask at 10 to 12 L/minute D. Notify the woman's health care provider

Choice B is correct; changing the woman to a lateral position will enhance cardiac output and raise her BP; because compression of the abdominal aorta and vena cava is removed, the other actions will follow; see Emergency—Maternal Hypotension with Decreased Placental Perfusion box.

Administering an opioid antagonist to a woman who is opioid dependent will result in the opioid abstinence syndrome. The nurse would recognize which clinical manifestations as evidence that this syndrome is occurring? (Circle all that apply.) A. Yawning B. Coughing C. Piloerection D. Anorexia E. Dry skin, eyes, and nose F. Miosis

Choices A, C, and D are correct; sneezing, rhinorrhea, sweating, lacrimation, and mydriasis also occur; see Signs of Potential Complications—Maternal Opioid Abstinence Syndrome box.

The nurse who performs vaginal examinations to assess a woman's progress in labor should: a. Perform an examination at least once every hour during the active phase of labor. b. Perform the examination with the woman in the supine position. c. Wear two clean gloves for each examination. d. Discuss the findings with the woman and her partner.

ANS:D The nurse should discuss the findings of the vaginal examination with the woman and her partner and report them to the primary care provider. A vaginal examination should be performed only when indicated by the status of the woman and her fetus. The woman should be positioned to avoid supine hypotension. The examiner should wear a sterile glove while performing a vaginal examination for a laboring woman.

You are evaluating the fetal monitor tracing of your client, who is in active labor. Suddenly you see the fetal heart rate (FHR) drop from its baseline of 125 down to 80. You reposition the mother, provide oxygen, increase intravenous (IV) fluid, and perform a vaginal examination. The cervix has not changed. Five minutes have passed, and the fetal heart rate remains in the 80s. What additional nursing measures should you take? a. Scream for help. b. Insert a Foley catheter. c. Start Pitocin. d. Notify the care provider immediately.

ANS:D To relieve an FHR deceleration, the nurse can reposition the mother, increase IV fluid, and provide oxygen. If oxytocin is infusing, it should be discontinued. If the FHR does not resolve, the primary care provider should be notified immediately. Inserting a Foley catheter is an inappropriate nursing action. If the FHR were to continue in a nonreassuring pattern, a cesarean section could be warranted, which would require a Foley catheter. However, the physician must make that determination. Pitocin may place additional stress on the fetus.

Maternity nurses often have to answer questions about the many, sometimes unusual ways people have tried to make the birthing experience more comfortable. For instance, nurses should be aware that: a. Music supplied by the support person has to be discouraged because it could disturb others or upset the hospital routine. b. Women in labor can benefit from sitting in a bathtub, but they must limit immersion to no longer than 15 minutes at a time. c. Effleurage is permissible, but counterpressure is almost always counterproductive. d. Electrodes attached to either side of the spine to provide high-intensity electrical impulses facilitate the release of endorphins.

ANS:D Transcutaneous electrical nerve stimulation does help. Music may be very helpful for reducing tension and certainly can be accommodated by the hospital. Women can stay in a bath as long as they want, although repeated baths with breaks may be more effective than a long soak. Counterpressure can help the woman cope with lower back pain.

In documenting labor experiences, nurses should know that a uterine contraction is described according to all these characteristics except: a. Frequency (how often contractions occur). b. Intensity (the strength of the contraction at its peak). c. Resting tone (the tension in the uterine muscle). d. Appearance (shape and height).

ANS:D Uterine contractions are described in terms of frequency, intensity, duration, and resting tone.


Conjuntos de estudio relacionados

Week 3 Quiz: Package and Software Management

View Set

Harry Potter and the Sorcerer's Stone

View Set

ch 5 merchandise business financial accounting

View Set

Chapter 49 Assessment and Management of Patients With Hepatic Disorders

View Set

Dialogue 80 - English French Anglais Français - Doing homework - Faire les devoirs

View Set